Download as pdf or txt
Download as pdf or txt
You are on page 1of 62

American Academy of Orthopaedic Surgeons

2001 Sports Medicine Self-Assessment Examination

1. An 18-year-old high school football player sustains a thigh injury that results in the
findings shown in Figure 1. Initial management should consist of

1- Gentle passive stretching.


2- Pulsed therapeutic ultrasonography.
3- Interferential electrical stimulation.
4- Cross-fiber friction massage.
5- Resting the muscle group.

PREFERRED RESPONSE: 5

DISCUSSION: The radiograph shows myositis ossificans within the quadriceps muscle. This
condition occurs as a complication of muscle injury. Initial treatment should include rest, ice,
compression, and elevation. While gentle active range of motion is encouraged in the functional
recovery from this injury, passive stretching is contraindicated as it can enhance hemorrhage and
accentuate the development of myositis ossificans. Ultrasound is similarly contraindicated
because it can enhance the development of myositis ossificans and has no proven efficacy in this
patient; electrical stimulation also has no proven benefits. Massage is contraindicated in the
initial management of this injury because of its influence on increasing local blood flow.

 Anderson JE (ed): Grant’s Atlas of Anatomy. Baltimore, MD, Williams & Wilkins, 1978, pp 4.39-4.49.
 Brumet ME, Hontas RB: The thigh, in DeLee JC, Drez D Jr (eds): Orthopaedic Sports Medicine. Philadelphia, PA, WB
Saunders, 1994, pp 1086-1112.
 Antao NA: Myositis of the hip in a professional soccer player: A case report. Am J Sports Med 1988;16:82-83.
 Jackson DW, Feagin JA: Quadriceps contusions in young athletes: Relation of severity of injury to treatment and prognosis.
J Bone Joint Surg Am 1973;55:95-105.

2. What is the function of the rotator cuff during throwing?

1- Limits humeral head translation in the transverse plane but not in the sagittal plane
2- Limits superior migration but not anterior and posterior translation
3- Limits superior migration and anterior and posterior translation
4- Provides little control of superior anterior and posterior translation
5- Creates inferior migration with maximal contraction during acceleration

PREFERRED RESPONSE: 3

DISCUSSION: The coupled action of the rotator cuff prevents superior migration and controls
anterior and posterior translation by depressing the humeral head.

 Poppen NK, Walker PS: Normal and abnormal motion of the shoulder. J Bone Joint Surg Am 1976;58:195-201.

Copyright©2001 by American Academy of Orthopaedic Surgeons™. All rights reserved.


36  American Academy of Orthopaedic Surgeons

 Abrams JS: Special shoulder problems in the throwing athlete: Pathology, diagnosis, and nonoperative management. Clin
Sports Med 1991;10:839-861.

3. A 24-year-old female soccer player has had lateral joint line pain and a recurrent
effusion in the left knee after sustaining a twisting injury 6 weeks ago. She reports
that symptoms worsen with athletic activities. MRI scans are shown in Figures 2a
through 2c. What is the most likely diagnosis?

1- Osteochondral fracture of the


lateral femoral condyle
2- Trabecular injury of the lateral
tibial plateau
3- Lateral meniscal tear with a
parameniscal cyst
4- Fibular collateral ligament tear
5- Discoid lateral meniscal tear

PREFERRED RESPONSE: 5

DISCUSSION: The MRI scans show the typical findings of a torn discoid lateral meniscus. The
average transverse diameter of the lateral meniscus is 11 or 12 mm. A discoid lateral meniscus is
suggested when three or more contiguous 5-mm sagittal sections on the MRI scan show
continuity of the menicus between the anterior and posterior horns, or when two adjacent
peripheral sagittal 5-mm sections show equal meniscal height. Normally the black “bow tie”
would be seen on two contiguous sagittal sections. The presence of a discoid meniscus can be
further confirmed if coronal views reveal increased width.

 Jordan MR: Lateral meniscal variants: Evaluation and treatment. J Am Acad Orthop Surg 1996;4:191-200.
 Resnick D, Kang HS: Internal Derangement of Joints: Emphasis on MRI Imaging. Philadelphia, PA, WB Saunders, 1997,
pp 625-630.
American Academy of Orthopaedic Surgeons
2001 Sports Medicine Self-Assessment Examination

4. A 29-year-old woman who underwent an anterior cruciate ligament (ACL)


reconstruction 6 months ago now reports difficulty achieving full knee extension,
and physical therapy fails to provide relief. The knee is stable on ligament testing.
Figure 3 shows the findings at a repeat arthroscopy. Treatment should now include

1- revision of the failing ACL reconstruction.


2- arthroscopic lysis of adhesions and
manipulation of the knee.
3- surgical removal of hypertrophic fibrous
tissue.
4- excision of the torn medial meniscus.
5- continued aggressive physical therapy.

PREFERRED RESPONSE: 3

DISCUSSION: The patient has a cyclops lesion. This is a nodule of fibroproliferative tissue that
originates from either drilling debris from the tibial tunnel or remnants of the ACL stump; more
rarely it is the result of broken graft fibers. The treatment of choice is excision of the nodule
and, if needed, additional notchplasty. Marked improvements in function and symptoms have
been noted after removal of the extension block and resumption of a rehabilitation program.

 Delince P, Krallis P, Descamps PY, et al: Different aspects of the cyclops lesion following anterior cruciate ligament
reconstruction: A multifactorial etiopathogenesis. Arthroscopy 1998;14:869-876.
 Fisher SE, Shelbourne KD: Arthroscopic treatment of symptomatic extension block complicating anterior cruciate ligament
reconstruction. Am J Sports Med 1993;4:558-564.

5. The major blood supply to the cruciate ligaments arises from which of the
following structures?

1- Superior genicular artery


2- Middle genicular artery
3- Inferior genicular artery
4- Infrapatellar fat pad
5- Intramedullary vessels

PREFERRED RESPONSE: 2

DISCUSSION: The major blood supply to the cruciate ligaments arises from the ligamentous
branches of the middle genicular artery. Few terminal branches of the inferior genicular artery
contribute to the blood supply. The synovial plexus and sheath covering the cruciate ligaments
are also supplied by branches of the middle genicular artery. The blood supply to the cruciate
ligaments is predominately of soft-tissue origin. There is no significant osseous vascular
contribution to the ligaments.

 Arnoczky SP: Anatomy of the anterior cruciate ligament. Clin Orthop 1983;172:19-25.
 Arnoczsky SP: Blood supply to the anterior cruciate ligament and supporting structures. Orthop Clin North Am 1985;16:15-
28.
Copyright©2001 by American Academy of Orthopaedic Surgeons™. All rights reserved.
38  American Academy of Orthopaedic Surgeons

6. In the anterior cruciate ligament (ACL)-deficient knee, which of the following


variables has the highest correlation with the development of arthritis?

1- Duration of time since the injury


2- Patient age
3- Additional ligament injury
4- Degree of laxity
5- Meniscal integrity

PREFERRED RESPONSE: 5

DISCUSSION: Ample evidence supports an increased rate of degenerative arthritis in the ACL-
deficient knee. Several variables play a role in the development of the arthritis, but the integrity
of the meniscus has been shown to be the single most important factor.

 O’Brien WR: Degenerative arthritis of the knee following anterior cruciate ligament injury: Role of the meniscus. Sports
Med Arthroscopy Rev 1993;1:114-118.
 Fetto JF, Marshall JL: The natural history and diagnosis of anterior cruciate ligament insufficiency. Clin Orthop
1980;147:29-38.
 McDaniel WJ Jr, Dameron TB Jr: The untreated anterior cruciate ligament rupture. Clin Orthop 1983;172:158-163.

7. A 20-year-old football player has immediate pain in the midfoot and is unable to
bear weight after an opposing player lands on the back of his plantar flexed foot.
AP and lateral radiographs are shown in Figures 4a and 4b. Management should
consist of :

1- closed reduction and a non-weight-bearing cast.


2- closed reduction and a weight-bearing cast.
3- closed reduction and percutaneous pinning.
4- open reduction and casting.
5- open reduction and internal fixation.
American Academy of Orthopaedic Surgeons
2001 Sports Medicine Self-Assessment Examination

PREFERRED RESPONSE: 5

DISCUSSION: The history and radiographs indicate a Lisfranc fracture-dislocation of the foot.
The radiographs show the classic “fleck sign,” which is an avulsion of the Lisfranc ligament
from the base of the second metatarsal. Most authors recommend open reduction and internal
fixation of this injury. Closed reduction can be attempted, but anatomic reduction is unlikely
because of the interposed bone fragments and soft tissues. Standard radiographs are not reliable
in identifying 1 to 2 mm of subluxation of the tarsometatarsal joint. The tarsometatarsal joint has
a poor tolerance to even mild subluxation, and the resulting decrease in joint contact area
increases the likelihood of posttraumatic arthritis. Open reduction with the joint visible allows
more anatomic reduction and internal fixation of larger osteochondral fragments or excision of
smaller interposed fragments.

 Bellabarba C, Sanders R: Dislocations of the foot, in Coughlin MJ, Mann RA (eds): Surgery of the Foot and Ankle, ed 7. St
Louis, MO, Mosby, 1999, vol 2, pp 1539-1558.
 Murphy GA: Fractures and dislocations of the foot, in Canale ST (ed): Campbell’s Operative Orthopaedics, ed 9. St Louis,
MO, Mosby, 1998, vol 2, pp 1956-1960.

8. What effect does deep freezing have on allograft tissue?

1- Causes no deleterious clinical effect on ligamentous grafts


2- Causes a less deleterious effect on cartilage than on ligamentous grafts
3- Causes degradation of the extracellular matrix
4- Allows for preservation of cells with tissue
5- Eliminates the chance of human immunodeficiency virus (HIV) transmission

PREFERRED RESPONSE: 1

DISCUSSION: Deep freezing is the simplest and most widely used method of ligament allograft
storage. All cells in the tissue are destroyed with the freezing. However, for this reason, it is not
a preferred storage method for menisci or cartilage allografts. Although this method may
enhance success because it removes potential antigens located on the cells, it cannot guarantee
elimination of HIV transmission. The advantage of cryopreservation storage is that a significant
number of cells will survive the process, a factor important in meniscal allograft survival after
implantation. No deleterious effects are noted clinically because of the acellularity of the tissue.

 Shelton WR, Treacy SH, Dukes AD, Bomboy AL: Use of allografts in
knee reconstruction: I. Basic science aspects and current status. J Am Acad Orthop Surg 1998;6:165-168.
 Caspari RB, Botherfield S, Horwitz RL, et al: HIV transmission via allograft organs and tissues. Sports Med Arthroscopy
Rev 1993;1:42-46.

9. A 32-year-old man who works as a laborer has had left trapezius wasting and lateral
scapular winging after injuring his shoulder when a cargo box fell onto his neck 8
Copyright©2001 by American Academy of Orthopaedic Surgeons™. All rights reserved.
40  American Academy of Orthopaedic Surgeons

months ago. He now reports posterior shoulder pain and fatigue, and he has
difficulty shrugging his shoulder. Examination reveals marked scapular winging,
impingement signs, and an asymmetrical appearance when the patient attempts a
shoulder shrug. Primary scapular-trapezius winging is the result of damage to the

1- spinal accessory nerve, causing shoulder depression with the scapula translated
laterally and the inferior angle rotated laterally.
2- spinal accessory nerve, causing shoulder elevation with the scapula translated
medially and the inferior angle rotated medially.
3- long thoracic nerve, causing shoulder elevation with the scapula translated
medially and the inferior angle rotated medially.
4- long thoracic nerve, causing shoulder depression with the scapula translated
laterally and the inferior angle rotated laterally.
5- thoracodorsal nerve, causing shoulder depression with the scapula translated
laterally and the inferior angle rotated laterally.

PREFERRED RESPONSE: 1

DISCUSSION: The patient has primary scapular-trapezius winging. This condition can be
caused by blunt trauma to the relatively superficial spinal accessory nerve that is located in the
floor of the posterior cervical triangle in the subcutaneous tissue. Other causes of injury include
penetrating trauma, traction, or surgical injury. With trapezius winging, the shoulder appears
depressed and laterally translated because of an unopposed serratus anterior. This contrasts with
primary serratus anterior winging, which is caused by injury to the long thoracic nerve. In this
condition, the scapula assumes a position of superior elevation and medial translation, and the
inferior angle is rotated medially. The thoracodorsal nerve supplies the latissimus dorsi and is
not involved in primary scapular winging.

 REFERENCES: Kuhn JE, Plancher KD, Hawkins RJ: Scapular winging. J Am Acad Orthop Surg 1995;3:319-325.
 Wright TA: Accessory spinal nerve injury. Clin Orthop 1975;108:15-18.

10. A 32-year-old football


coach has had a 4-month
history of increasing right
wrist pain, particularly
during blocking exercises,
and he reports significant
pain with range of motion
and gripping activities.
He denies any history of
trauma. Examination
reveals dorsal wrist
tenderness and boggy
fullness over the dorsum
of the wrist. No erythema
is noted. Grip strength is
60% compared with the opposite side. Radiographs are shown in Figures 5a and
5b. What is the most likely diagnosis?
American Academy of Orthopaedic Surgeons
2001 Sports Medicine Self-Assessment Examination

1- Scapholunate dissociation
2- Triangular fibrocartilage tear
3- Scaphoid fracture
4- Perilunate dislocation
5- Kienbock’s disease

PREFERRED RESPONSE: 5

DISCUSSION: The patient has Kienbock’s disease (osteonecrosis of the lunate), which presents
with boggy synovitis of the wrist, decreased range of motion, and often normal radiographs. The
patient’s radiographs reveal small fragments from the lunate, with increased density in the lunate
body. While a traumatic event may precede the patient’s pain, often an insidious increase in pain
is found. Repetitive trauma has been suggested as a possible cause. This disease process is
classically associated with an ulnar-negative variant. An MRI scan, revealing a low-intensity
signal in the lunate, is the best diagnostic tool for early Kienbock’s disease.

 Green DP, Hotchkiss RN, Pederson WC: Green’s Operative Hand Surgery, ed 4. Philadelphia, PA, Churchill Livingstone,
1999, pp 837-848.
 Gerwin M, Weiland AJ: Avascular necrosis of the carpals. Hand Clin 1993, p 761.

11. Which of the following properties apply to the human meniscus when compared
with articular cartilage?

1- Less elastic and less permeable


2- Less elastic and more permeable
3- Of the same elasticity and permeability
4- More elastic and more permeable
5- More elastic and less permeable

PREFERRED RESPONSE: 5

DISCUSSION: The meniscal cartilage, like articular cartilage, possesses viscoelastic properties.
The extracellular matrix is a biphasic structure composed of a solid phase (collagen,
proteoglycan) that acts as a fiber-reinforced porous-permeable composite, and a fluid phase that
may be forced through the solid matrix by a hydraulic pressure gradient. Although these
properties are shared with articular cartilage, the meniscus is more elastic and less permeable
than articular cartilage.

 Beaty JH (ed): Orthopaedic Knowledge Update 6. Rosemont, IL, American Academy of Orthopaedic Surgeons, 1999, pp 3-
23.
 Arendt EA (ed): Orthopaedic Knowledge Update: Sports Medicine 2. Rosemont, IL, AAOS, 1999, pp 349-354.

12. An 18-year-old football player lands on a flexed knee and ankle after being tackled.
Examination reveals increased external rotation and posterior translation and varus
Copyright©2001 by American Academy of Orthopaedic Surgeons™. All rights reserved.
42  American Academy of Orthopaedic Surgeons

at 30° of flexion, which decreases as the knee is flexed to 90°. What is the most
likely diagnosis?

1- Torn posterolateral corner


2- Torn posterior cruciate ligament (PCL) and posterolateral corner
3- Torn PCL
4- Rupture of the quadriceps tendon
5- Rupture of the lateral collateral ligament

PREFERRED RESPONSE: 1

DISCUSSION: The flexed knee and ankle mechanism of injury can result in a PCL and/or
posterolateral corner injury. The examination reveals an isolated injury to the posterolateral
corner (arcuate, popliteus, posterolateral capsule). This results in increased posterior translation
and external rotation, as well as varus that is most notable at 30° of flexion and decreases as the
knee is further flexed to 90°. Combined PCL and posterolateral corner injuries are characterized
by increasing instability as the knee is flexed to 90° from 30°, while isolated PCL tears show the
greatest degree of instability at 90° of flexion. A rupture of the quadriceps tendon would not
affect anterior or posterior stability, whereas an isolated rupture of the lateral collateral ligament,
which is a rare injury, is characterized by varus instability at 30° of knee flexion without
posterior translation.

 Harner CD, Hoher J: Evaluation and treatment of posterior cruciate ligament injuries. Am J Sports Med 1998;26:471-482.
 Veltri DM, Warren RF: Isolated and combined posterior cruciate ligament injuries. J Am Acad Orthop Surg 1993;1:67-75.

13. Figure 6 shows the radiograph of a 14-year-old baseball player who felt a pop and
had an immediate onset of pain in his elbow after a hard throw from the outfield.
The best course of action should be to

1- obtain stress radiographs of the elbow.


2- obtain an MRI scan of the elbow.
3- apply a splint and initiate early range-of-motion
exercises.
4- apply a cast in 90° of flexion for 4 weeks.
5- perform open reduction and internal fixation.

PREFERRED RESPONSE: 5

DISCUSSION: The valgus stress at the elbow caused by throwing strains the medial collateral
ligament. The medial epicondyle, on which the ligament inserts, is the last ossification center to
fuse to the distal humerus, and acute avulsion of the medial epicondyle can occur in adolescents.
If the elbow is allowed to heal in a displaced position, valgus instability and loss of elbow
extension may result. Valgus instability is especially problematic for the throwing athlete.
American Academy of Orthopaedic Surgeons
2001 Sports Medicine Self-Assessment Examination

Surgical treatment with rigid internal fixation is the treatment of choice for displaced medial
epicondyle avulsion fractures. Valgus instability is prevented, and the rigid fixation allows for
early range of motion.

 Case SL, Hennrikus WL: Surgical treatment of displaced medial epicondyle fractures in adolescent athletes. Am J Sports
Med 1997;25:682-686.
 Woods GW, Tullos HS: Elbow instability and medial epicondyle fractures. Am J Sports Med 1977;5:23-30.

14. Osteophyte formation at the posteromedial olecranon and olecranon articulation in


high-caliber throwing athletes is most often the result of underlying

1- anterior capsular tears.


2- forearm pronator and flexor muscle weakness.
3- biceps or brachialis muscle weakness.
4- ulnar collateral ligament insufficiency.
5- radial collateral ligament insufficiency.

PREFERRED RESPONSE: 4

DISCUSSION: During the late acceleration phase of throwing, the triceps forcibly contracts,
extending the elbow as the ball is released. Normally, this force is absorbed by the anterior
capsule and the brachialis and biceps muscles. However, if the ulnar collateral ligament is
insufficient, the elbow will be in a subluxated position during extension and cause impaction of
the olecranon and the olecranon fossa posteromedially. Over time, osteophyte formation is
likely to occur.

 Conway JE, Jobe FW, Glousman RE, Pink M: Medial instability of the elbow in throwing athletes: Treatment by repair or
reconstruction of the ulnar collateral ligament. J Bone Joint Surg Am 1992;74:67-83.
 Wilson FD, Andrews, JR, Blackburn TA, McCluskey G: Valgus extension overload in the pitching elbow. Am J Sports
Med 1983;11:83-88.

15. Sudden cardiac death in the young athlete is most frequently caused by

1- hypertrophic cardiomyopathy.
2- active myocarditis.
3- mitral valve prolapse.
4- aortic rupture.
5- coronary artery disease.

PREFERRED RESPONSE: 1

DISCUSSION: Hypertrophic cardiomyopathy is the leading cause of sudden cardiac death in


athletes, accounting for 40% of reported cases. Most athletes have no previous symptoms, and
sudden death may be the first clinical manifestation. The prevalence of hypertrophic
cardiomyopathy in the general population is 1 in 500, with a mortality rate of 2% to 4% in young
Copyright©2001 by American Academy of Orthopaedic Surgeons™. All rights reserved.
44  American Academy of Orthopaedic Surgeons

adults. Athletes with active myocarditis should not engage in sports for up to 6 months, and
although they may be at risk for the development of chronic cardiomyopathy, it is rarely a cause
of sudden cardiac death. Mitral valve prolapse with an accompanying systolic murmur is
common in the general population, but infrequently a cause of sudden cardiac death. Weakening
of the aortic wall associated with Marfan syndrome can result in abrupt rupture of the aorta. This
accounts for 3% of sudden cardiac deaths in young athletes. Marfan syndrome usually can be
detected on preparticipation screenings by its skeletal and ocular manifestations. Atherosclerotic
coronary artery disease is the most common cause of sudden cardiac death in older athletes,
accounting for 75% of reported cases. However, it is much less common in the young
competitive athlete.

 Burke AP, Farb A, Virmani R, Goodin J, Smialek JE: Sports-related and non-sports-related sudden cardiac death in young
adults. Am Heart J 1991;121:568-575.
 Maron BJ, Sharani J, Poliac LC, Mathenge R, Roberts WC, Mueller FO: Sudden death in young competitive athletes:
Clinical, demographic, and pathological profiles. JAMA
1996;276:199-204.

16. A 14-year-old football player has had right knee pain for the past 2 months;
however, he denies any history of trauma. Examination shows an abductor lurch
and increased external rotation of the right lower extremity. The best course of
action should be to

1- apply a knee sleeve during sports.


2- withdraw from football for 2 weeks.
3- obtain AP and frog-lateral radiographs of the pelvis.
4- obtain an MRI scan of the right knee.
5- initiate physical therapy.

PREFERRED RESPONSE: 3

DISCUSSION: Slipped capital femoral epiphysis is the most common pathology involving the
hip in adolescents. While patients with acute slips may report severe pain and are unable to
ambulate, those with chronic slips often have pain during ambulation, a limp, and increased
external rotation of the hip. While 60% of the patients specifically report hip pain, the remainder
have pain in the thigh or knee. The initial diagnostic study of choice is AP and frog-lateral
radiographs of the pelvis; bilateral involvement is frequently seen.

 Boyer DW, Mickelson MR, Ponseti IV: Slipped capital femoral epiphysis: Long-term follow-up study of one hundred and
twenty-one patients. J Bone Joint Surg Am 1981;63:85-95.
 Stasikelis PJ, Sullivan CM, Philips WA, Polard JA: Slipped capital femoral epiphysis: Prediction of contralateral
involvement. J Bone Joint Surg Am 1996;78:1149-1155.

17. Which of the following is considered the appropriate initial management protocol
for an unconscious football player without spontaneous respirations?
American Academy of Orthopaedic Surgeons
2001 Sports Medicine Self-Assessment Examination

1- Log roll to a supine position, remove the helmet, and begin assisted breathing
2- Stabilize the head and neck, log roll to a supine position, remove the helmet, and
begin assisted breathing
3- Log roll to a supine position, stabilize the head and neck, remove the face mask,
and begin cardiopulmonary respiration (CPR)
4- Log roll onto a spine board, stabilize the head and neck, remove the face mask,
and begin CPR
5- Stabilize the head and neck, log roll to a supine position, remove the face mask,
and begin assisted breathing

PREFERRED RESPONSE: 5

DISCUSSION: The on-field evaluation and management of the seriously injured athlete requires
advance preparation and planning. It is imperative that the health care team have a game plan in
place and the proper equipment readily available. The initial step consists of stabilizing the head
and neck by manually holding the head and neck in a neutral position. Then, in the following
order, check for breathing, pulses, and level of consciousness. If the athlete is breathing, simply
remove the mouth guard and maintain the airway. If the athlete is not breathing, the face mask
must be removed and the chin strap left in place. An open airway must be established, followed
by assisted breathing. CPR is only instituted when breathing and circulation are compromised.
If the athlete is unconcious or has a suspected cervical spine injury, the helmet must not be
removed until the athlete has been transported to an appropriate facility and the cervical spine
has been completely evaluated.

 McSwain NE, Garnelli RL: Helmet removal from injured patients. Bull Am Coll Surg 1997;82:42-44.
 Vegso JJ, Lehman RC: Field evaluation and management of head and neck injuries. Clin Sports Med 1987;6:1-15.
 Arndt EA (ed): Orthopaedic Knowledge Update: Sports Medicine 2. Rosemont, IL, American Academy of Orthopaedic
Surgeons, 1999, pp 93-101.

18. Figure 7 shows the radiograph of an 18-year-old hockey player who sustained a
shoulder injury during a fall into the side boards. Examination reveals a significant
prominence at the acromioclavicular joint. Management should consist of

1- a figure-of-8 clavicle strap.


2- a sling for comfort, followed by early
range-of-motion and strengthening
exercises.
3- open reduction and stabilization.
4- immobilization in a spica cast.
5- resection of the distal clavicle.

PREFERRED RESPONSE: 3

DISCUSSION: The radiograph shows a type V acromioclavicular separation with greater than
100% superior elevation of the clavicle. This finding implies detachment of the deltoid and
trapezius from the distal clavicle. Because of severe compromise of function and potential
compromise to the overlying skin, surgery is the treatment of choice for type V
Copyright©2001 by American Academy of Orthopaedic Surgeons™. All rights reserved.
46  American Academy of Orthopaedic Surgeons

acromioclavicular separations. During reduction and repair, meticulous repair of the


deltotrapezial fascia will also aid in securing the repair.

 Nuber GW, Bowen MK: Acromioclavicular joint injuries and distal clavicle fractures. J Am Acad Orthop Surg 1997;5:11-
18.
 Weinstein DM, McCann PD, McIlveen SJ, Flatow EL, Bigliani LU: Surgical treatment of complete acromioclavicular
dislocations. Am J Sports Med 1995;23:324-331.

19. A 22-year-old professional ballet dancer reports a 3-month history of posterior


ankle pain that occurs when she changes from a flat foot to pointe (hyperplantar
flexed position). Examination does not elicit the pain with forced passive plantar
flexion. A radiograph is shown in Figure 8. What is the most likely cause of the
pain?

1- Mild subtalar arthritis


2- Posterior tibialis tendinitis
3- Os trigonum entrapment syndrome
4- Flexor hallucis longus tenosynovitis
5- Retrocalcaneal bursitis

PREFERRED RESPONSE: 4

DISCUSSION: The most common causes of posterior ankle pain in ballet dancers are flexor
hallucis longus tenosynovitis and os trigonum syndrome. Flexor hallucis longus tenosynovitis
differs from a symptomatic os trigonum by the absence of pain with forced plantar flexion and
the presence of pain with resisted plantar flexion of the great toe. The pain is often felt in the
posterior ankle and can be associated with a snapping or triggering sensation. Os trigonum
syndrome commonly occurs in ballet dancers who perform in a position of extreme plantar
flexion. The pain occurs from entrapment of the os trigonum between the posterior portion of
the talus and calcaneus.

 Hamilton WG, Geppert MJ, Thompson FM: Pain in the posterior aspect of the ankle in dancers: Differential diagnosis and
operative treatment. J Bone Joint Surg Am 1996;78:1491-1500.
 Khan K, Brown J, Way S, et al: Overuse injuries in classical ballet. Sports Med
1995;19:341-357.

20. An 18-year-old man recently underwent an uncomplicated arthroscopic partial


medial meniscectomy that was complicated by reflex sympathetic dystrophy (RSD),
also termed “sympathetically maintained pain” (SMP). What is the most common
finding of this condition?

1- Joint stiffness
2- Cold intolerance
American Academy of Orthopaedic Surgeons
2001 Sports Medicine Self-Assessment Examination

3- Decreased sweating
4- Osteopenia
5- Disproportionate pain

PREFERRED RESPONSE: 5

DISCUSSION: The hallmark for RSD or SMP is the presence of pain that is out of proportion to
that expected for the degree of the injury. SMP often extends well beyond the involved area and
is present in a nonanatomic distribution. The pain is frequently described as a burning sensation,
with extreme sensitivity to light touch. Joint stiffness can be present but is a nonspecific finding.
There may be cold intolerance, but this is not a cardinal symptom. Sweating actually may be
increased. Osteopenia, if present, is a late finding.

 Lindenfeld TN, Bach BR Jr, Wojtys EM: Reflex sympathetic dystrophy and pain dysfunction in the lower extremity. Instr
Course Lect 1997;46:261-268.
 O’Brien SJ, Ngeow J, Gibney MA, Warren RF, Fealy S: Reflex sympathetic dystrophy of the knee: Causes, diagnosis, and
treatment. Am J Sports Med 1995;23:655-659.

21. What is the main function of collagen found within articular cartilage?

1- Compressive properties
2- Tensile properties
3- Proteoglycan synthesis
4- Cartilage metabolism
5- Joint lubrication

PREFERRED RESPONSE: 2

DISCUSSION: The main function of collagen in articular cartilage is to provide the tissue’s
tensile strength. It also immobilizes proteoglycans within the extracellular matrix. Compressive
properties are maintained by proteoglycans. Cartilage metabolism is maintained by the
indwelling chondrocytes. The flow of water through the tissue promotes transport of nutrients
and provides a source of lubricant for the joint.

 Simon SR (ed): Orthopaedic Basic Science. Rosemont, IL, American Academy of Orthopaedic Surgeons, 1994, pp 3-44.
 Mow VC, Ratcliffe A: Structure and function of articular cartilage and meniscus, in Mow VC, Hayes WC (eds): Basic
Orthopaedic Biomechanics, ed 2. Philadelphia, PA, Lippincott-Raven, 1997, pp 113-177.

22. A 15-year-old girl who competes in gymnastics has immediate pain and giving way
of the left elbow after falling from the uneven parallel bars and landing on her
outstretched arms. Examination reveals swelling and tenderness about the elbow,
especially over the medial side. Measurement of elbow motion shows 0° to 125° of
flexion, and valgus stress at the elbow is painful. AP, lateral, and stress radiographs
are shown in Figures 9a through 9c. Management should consist of

Copyright©2001 by American Academy of Orthopaedic Surgeons™. All rights reserved.


48  American Academy of Orthopaedic Surgeons

1- arthroscopic repair of the ulnar collateral ligament.


2- direct surgical repair of the ulnar collateral ligament.
3- reconstruction of the ulnar collateral ligament with a palmaris longus tendon
autograft.
4- a hinged elbow brace to allow early protected range of motion.
5- immobilization of the elbow to allow healing of the ulnar collateral ligament.

PREFERRED RESPONSE: 3

DISCUSSION: While many low-demand patients with injuries to the ulnar collateral ligament
can be treated nonsurgically, Jobe and associates described two situations in which ulnar
collateral ligament reconstruction is indicated: (1) an acute complete rupture in a competitive
athlete who uses the upper extremities extensively and who wishes to remain active; and (2)
chronic pain or instability that does not improve after at least 3 months of nonsurgical
management. Rarely is direct surgical repair of the ligament possible or able to withstand the
valgus stresses applied to the elbow. Most authors recommend surgical reconstruction of the
ulnar collateral ligament using a palmaris longus, plantaris, or fourth toe extensor tendon from
the fourth autograft.

 Andrews JR, Jelsma RD, Joyce ME, et al: Open surgical procedures for injuries to the elbow in throwers. Oper Tech Sports
Med 1994;4:109-133.
 Jobe FW, Kvitne RS: Elbow instability in the athlete. Instr Course Lect 1991;40:17-23.
 Smith GR, Altchek DW, Pagnani MJ, Keeley JR: A muscle-splitting approach to the ulnar collateral ligament of the elbow:
Neuroanatomy and operative technique. Am J Sports Med 1996;24:575-580.

23. A 15-year-old boy who participates in track reports acute pain along the left iliac
crest during a sprint. Examination reveals that the anterior superior iliac spine is
nontender. The most likely diagnosis is an injury to the

1- epiphysis.
2- apophysis.
3- enthesis.
American Academy of Orthopaedic Surgeons
2001 Sports Medicine Self-Assessment Examination

4- tendon.
5- muscle.

PREFERRED RESPONSE: 2

DISCUSSION: The patient has iliac apophysitis. The radiographic findings are easily
overlooked but usually reveal slight asymmetric widening of the iliac crest apophysis. The
apophysis is the most vulnerable structure, as it is three to five times weaker than the tendon.
This is not an epiphyseal site, and injury to the muscle or the tendinous insertion to bone
(enthesis) is unlikely.

 Clancy WG Jr, Foltz AS: Iliac apophysitis and stress fractures in adolescent runners. Am J Sports Med 1976;4:214-218.
 Waters PM, Millis MB: Hip and pelvic injuries in the young athlete, in Stanitski CL, DeLee JC, Drez D Jr (eds): Pediatric
and Adolescent Sports Medicine. Philadelphia, PA, WB Saunders, 1994, pp 279-293.
 Lombardo SJ, Retting AC, Kerlan RK: Radiographic abnormalities of the iliac apophysis in adolescent athletes. J Bone
Joint Surg Am 1983;65:444-446.
 Paletta GA Jr, Andrish JT: Injuries about the hip and pelvis in the young athlete. Clin Sports Med 1995;14:591-628.

24. A 40-year-old woman who is an avid tennis player reports the insidious onset of
progressive left shoulder pain for the past 2 months. Examination reveals full range
of motion with a positive impingement sign. Strength in the supraspinatus and
infraspinatus muscles is normal, although stress testing is painful. An earlier
subacromial cortisone injection provided good, but only temporary relief. An AP
radiograph of the left shoulder is shown in Figure 10. Management should now
consist of

1- a rotator cuff exercise program and anti-


inflammatory drugs.
2- repeat subacromial cortisone injections as
necessary.
3- open subacromial decompression.
4- arthroscopic evacuation of calcium
deposits.
5- open rotator cuff repair.

PREFERRED RESPONSE: 4

DISCUSSION: The radiograph shows calcific deposits within the substance of the supraspinatus
tendon. Patients with this condition are prone to recurrent bouts of acute inflammation in the
shoulder. While the response to cortisone injection is often dramatic, repeated injections are not
recommended because of injury to the collagen fibers. Good results have been obtained with
arthroscopic evacuation of the calcium deposits. In one study, the addition of a subacromial
decompression did not improve the results.
Copyright©2001 by American Academy of Orthopaedic Surgeons™. All rights reserved.
50  American Academy of Orthopaedic Surgeons

 Jerosch J, Strauss JM, Schmiel S: Arthroscopic treatment of calcific tendinitis of the shoulder. J Shoulder Elbow Surg
1998;7:30-37.
 Ark JW, Flock TJ, Flatow EL, Bigliani LU: Arthroscopic treatment of calcific tendinitis of the shoulder. Arthroscopy
1992;8:183-188.

25. Which of the following nerves is susceptible to entrapment near the calcaneal
attachment site of the plantar fascia and can mimic or co-exist with plantar
fasciitis?

1- First branch of the lateral plantar nerve


2- Dorsal cutaneous branch of the superficial peroneal nerve
3- Medial calcaneal branch of the posterior tibial nerve
4- Lateral branch of the medial plantar nerve
5- Communicating branch of the fourth common digital nerve

PREFERRED RESPONSE: 1

DISCUSSION: The first branch of the lateral plantar nerve is susceptible to entrapment beneath
the deep fascia of the adductor hallucis muscle adjacent to the calcaneal attachment of the plantar
fascia. This can be a cause of chronic heel pain. Additionally, the nerve is vulnerable to injury
by a blind dissection in releasing the plantar fascia. The dorsal cutaneous branch of the
superficial peroneal nerve supplies sensation to the dorsum of the foot. The medial calcaneal
branch of the posterior tibial nerve lies in the subcutaneous tissues and innervates the skin of the
heel. It is vulnerable to injury from skin incisions on the medial side of the heel. The lateral
branch of the medial plantar nerve forms the second and third common digital nerves.
Entrapment of the proper medial plantar nerve can occur at the master knot of Henry. This is
well distal to the calcaneal attachment of the plantar fascia, and the pain usually radiates more
distally in the arch, separate from heel pain. The communicating branch of the fourth common
digital nerve crosses to the third common digital nerve. Therefore, the third common digital
nerve receives supply from both the lateral and medial plantar nerves. This dual supply has been
implicated in the increased incidence of digital neuroma of the third common digital nerve.

 Bordelon RL: Heel pain, in Mann RA, Coughlin MJ (eds): Surgery of the Foot and Ankle, ed 6. St Louis, MO, CV Mosby,
1993, pp 837-857.
 Mann RA, Baxter DE: Diseases of the nerves, in Mann RA, Coughlin MJ (eds): Surgery of the Foot and Ankle, ed 6. St
Louis, MO, CV Mosby, 1993, pp 543-574.
 Baxter DE: The heel in sport. Clin Sports Med 1994;13:683-693.

26. Figure 11 shows the radiograph of an 18-year-old soccer player who reports
recurrent lateral foot pain after sustaining an inversion injury. History reveals that
6 months ago he had been treated in a non-weight-bearing cast for a fifth metatarsal
fracture. Management should consist of
American Academy of Orthopaedic Surgeons
2001 Sports Medicine Self-Assessment Examination

1- intermedullary fixation.
2- a brace or taping to limit inversion stress.
3- a short leg walking cast or a fracture walker.
4- a non-weight-bearing short leg cast.
5- a rigid orthotic insole, with early motion
exercises.

PREFERRED RESPONSE: 1

DISCUSSION: Fractures in this area of the fifth metatarsal have a high incidence of delayed
union, nonunion, and recurrence with nonsurgical management. In an acute fracture, prolonged
casting in a non-weight-bearing cast may allow for healing; however, in the presence of
prolonged symptoms, recurrent fracture, and intermedullary sclerosis, surgical treatment is
preferred. Surgery most commonly consists of intermedullary fixation or medullary curettage
and bone grafting, followed by application of a non-weight-bearing cast.

 Torg JS, Balduini FC, Zelko RR, Pavlov H, Peff TC, Das M: Fractures of the base of the fifth metatarsal distal to the
tuberosity: Classification and guidelines for nonsurgical and surgical management. J Bone Joint Surg Am 1984;66:209-214.
 DeLee JC: Fractures and dislocations of the foot, in Mann R, Coughlin M (eds): Surgery of the Foot and Ankle, ed 6. St
Louis, MO, Mosby, 1993, pp 1465-1503.

27. Which of the following types of exercise used to increase flexibility is considered
most beneficial in increasing joint range of motion?

1- Ballistic stretching
2- Static stretching
3- Proprioceptive neuromuscular facilitation (PNF)
4- Isokinetic
5- Eccentric

PREFERRED RESPONSE: 3

DISCUSSION: Evidence has shown that PNF is the treatment of choice to increase joint range of
motion and flexibility. PNF has the advantage of pushing the patient to stretch a little further
when the muscle tendon unit is relaxed by a partner. While isokinetic and eccentric exercises
can improve flexibility, and therefore increase range of motion, their main purpose is to increase
strength and endurance. Ballistic stretching involves a large load applied rapidly; however,
evidence has shown that static stretching, where a low load is applied for a long duration, offers
a more significant benefit.
Copyright©2001 by American Academy of Orthopaedic Surgeons™. All rights reserved.
52  American Academy of Orthopaedic Surgeons

 Sady SP, Wortman M, Blanke D: Flexibility training: Ballistic, static or proprioceptive neuromuscular facilitation? Arch
Phys Med Rehabil 1982;63:261-263.
 Tanigawa MC: Comparison of the hold-relax procedure and passive mobilization on increasing muscle length. Phys Ther
1972;52:725-735.
 Wallin D, Ekblom B, Grahn R, Nordenberg T: Improvement of muscle flexibility: A comparison between two techniques.
Am J Sports Med 1985;13:263-268.

28. The view from an anterosuperior portal of the right shoulder shown in Figure 12
reveals which of the following findings?

1- Rupture of the subscapularis tendon


2- Tear of the rotator interval
3- Humeral avulsion of the glenohumeral
ligament (HAGL) lesion
4- Anterior ligamentous periosteal sleeve
avulsion (ALPSA) lesion
5- Bankart lesion

PREFERRED RESPONSE: 3

DISCUSSION: The arthroscopic view shows a HAGL lesion. With the arthroscope directed
anteroinferiorly, muscular striations of the subscapularis can be visualized through the avulsion
site. In vitro strain studies indicate that glenohumeral ligament failure on the humeral side
occurs in approximately 25% of patients, while clinically this lesion has been reported in
approximately 9% of patients with shoulder instability. Failure to recognize and treat this lesion
leads to persistent anterior instability. An ALPSA lesion, a Bankart variant, occurs on the
glenoid side and is characterized by a sleeve-like medial retraction and inferior rotation. A
Bankart lesion is the classic avulsion of the glenohumeral ligament from the glenoid rim. The
subscapularis tendon and the rotator interval are not shown in the figure.

 Wolf EM, Cheng JC, Dickson K: Humeral avulsion of glenohumeral ligaments as a cause of anterior shoulder instability.
Arthroscopy 1995;11:600-607.
 Bigliani LU, Pollack RG, Soslowsky LJ, Flatow EL, Pawluk RJ, Mow VC: Tensile properties of the inferior glenohumeral
ligament. J Orthop Res 1992;10:187-197.
 Warner JJ, Beim GM: Combined Bankart and HAGL lesion associated with anterior shoulder instability. Arthroscopy
1997;13:749-752.

29. An 18-year-old football player has intense pain and is unable to bear weight on the
right knee after being tackled from the front. A posterior knee dislocation is
reduced on the field. Because the game took place in a remote location, the patient
is not examined in the emergency department until 5 hours after the injury.
Examination now shows a grossly swollen knee with moderate ischemia in the lower
leg. Posterior tibial and dorsalis pedis pulses are diminished. The best course of
action should be to

1- obtain an emergent arteriogram.


American Academy of Orthopaedic Surgeons
2001 Sports Medicine Self-Assessment Examination

2- obtain an emergent MRI scan.


3- perform a thorough examination of the knee ligaments.
4- perform surgical repair or bypass of the injured popliteal vessels.
5- perform surgical repair or bypass of the injured popliteal vessels and ligament
reconstruction.

PREFERRED RESPONSE: 4

DISCUSSION: Vascular injuries occur in approximately 20% to 35% of knee dislocations, of


which one third are posterior. Recognition of the vascular injury is essential. Normal pulses or
normal capillary refill do not preclude an arterial injury, and arteriography should be considered
in all knee dislocations. If the leg is ischemic, the arteriogram should be circumvented and the
patient taken directly to the operating room. The risk of muscle fibrosis, contracture, or vascular
insufficiency, and the need for amputation increase significantly when ischemia exceeds 6 hours.
This patient has ischemia and is considered a vascular emergency. As such, delays for a
thorough examination of the ligament, MRI scans, and even an arteriogram are unwarranted.
Concurrent ligamentous repair and reconstruction should be deferred until vascular stability has
been achieved.

 Kremchek TE, Welling RE, Kremchek EJ: Traumatic dislocation of the knee. Orthop Rev 1989;18:1051-1057.
 Reckling FW, Peltier LF: Acute knee dislocations and their complications. J Trauma 1969;9:181-191.

30. A 17-year-old football player is unable to flex the distal interphalangeal (DIP) joint
of his ring finger. He states that he injured the finger 6 weeks ago while attempting
to tackle another player who pulled free from his grip, but he did not inform his
coach at the time of the injury. Current radiographs show an observable fleck of
bone volar to the base of the proximal phalanx. Treatment should consist of

1- fusion of the DIP joint with no reconstruction of the tendon.


2- advancement and repair of the tendon to the base of the distal phalanx.
3- two-stage reconstruction of the profundus tendon.
4- Z-plasty advancement of the profundus tendon.
5- tenodesis of the distal tendon remnant with the flexor digitorum sublimis.

PREFERRED RESPONSE: 2

DISCUSSION: Flexor digitorum profundus ruptures are classified into three types. In type I, the
tendon retracts into the palm. In type II, the tendon retracts to the level of the proximal phalanx,
the vinculum remains intact, and the blood supply is preserved to the tendon. A small fleck of
bony fragment observed at the A2 pulley is pathognomonic for a type II rupture. Successful
primary repair of the type II rupture has been reported as late as 2 months after the injury. Type
III injuries have large fragments of the distal phalanx attached and are caught distally by the A1
pulley. Type III ruptures can be repaired up to several months after the injury.

 Leddy JP: Avulsions of the flexor digitorum profundus. Hand Clin 1985;1:77-83.
 Kiefhaber TR: Closed tendon injuries in the hand. Oper Tech Sports Med 1996;4:227-241.

Copyright©2001 by American Academy of Orthopaedic Surgeons™. All rights reserved.


54  American Academy of Orthopaedic Surgeons

31. A 48-year-old ski instructor dislocates his nondominant shoulder in a fall.


Management consisting of application of a sling for 1 week results in improvement
in his pain. Follow-up examination 6 weeks after the injury reveals that the patient
continues to have difficulty with shoulder elevation. Management should now
include
1- use of the sling for an additional 3 weeks.
2- physical therapy.
3- a corticosteroid injection.
4- an MRI scan of the rotator cuff.
5- arthroscopic labral repair.

PREFERRED RESPONSE: 4

DISCUSSION: Patients who are older than age 45 years and have initial dislocations are at
greater risk for tearing the rotator cuff. Patients who are unable to lift the upper extremity or
who have continued pain should undergo further evaluation for potential rotator cuff tears; early
diagnosis is preferred. Physical therapy or continued use of a sling will be of little benefit. A
corticosteroid injection might delay the diagnosis and compromise subsequent rotator cuff repair.
Repairing the labrum generally is not necessary in a patient of this age who has an initial
dislocation.
 Hawkins RJ, Bell RH, Hawkins RH, Koppert GJ: Anterior dislocation of the shoulder in the older patient. Clin Orthop
1986;206:192-195.
 Matsen FA III, Thomas SC, Rockwood CA: Anterior glenohumeral instability, in Rockwood CA, Matsen FA III (eds): The
Shoulder. Philadelphia, PA, WB Saunders, 1990, pp 526-622.

32. Figure 13 shows the MRI scan of a 29-year-old rock climber who reports increasing
shoulder pain and weakness. Based on these findings, atrophy will most likely occur
in which of the following muscles?

1- Infraspinatus and supraspinatus


2- Infraspinatus
3- Supraspinatus
4- Teres minor
5- Deltoid

PREFERRED RESPONSE: 2

DISCUSSION: The MRI scan shows a cyst at the spinoglenoid notch. These cysts are often
associated with a labral injury, such as a superior labrum anterior and posterior (SLAP) lesion.
The suprascapular nerve passes through the suprascapular notch and sends motor branches to the
supraspinatus and sensory branches to the capsule. At the spinoglenoid notch, the infraspinatus
American Academy of Orthopaedic Surgeons
2001 Sports Medicine Self-Assessment Examination

branch of the suprascapular nerve is compressed by the cyst, leading to isolated infraspinatus
atrophy. The teres minor and the deltoid are innervated by the axillary nerve.
 Fehrman DA, Orwin JF, Jennings RM: Suprascapular nerve entrapment by ganglion cysts: A report of six cases with
arthroscopic findings and review of the literature. Arthroscopy 1995;11:727-734.
 Ianotti JP, Ramsey ML: Arthroscopic decompression of a ganglion cyst causing suprascapular nerve compression.
Arthroscopy 1996;12:739-745.
 Tirman PF, Feller JF, Janzen DL, Peterfy CG, Bergman AG: Association of glenoid labral cysts and labral tears in
glenohumeral instability: Radiologic findings and clinical significance. Radiology 1994;190:653-658.

33. A 46-year-old man has acute tenderness along the ulnar aspect of the wrist after
falling on his outstretched hand while playing basketball. Examination reveals
tenderness and mild swelling along the volar ulnar aspect of the wrist. Radiogaphs
are shown in Figures 14a through 14c. Management should consist of

1- immobilization.
2- closed reduction.
3- open reduction and internal fixation.
4- early range of motion.
5- excision.

PREFERRED RESPONSE: 1

DISCUSSION: The PA view of the wrist shows a pisiform fracture. Pisiform fractures
constitute 1% to 3% of all carpal bone fractures. This fracture can be further evaluated with a
carpal tunnel view or a supination oblique view of the wrist. Initial management should consist
of immobilization with a short arm cast. If nonsurgical measures fail, bony excision is
warranted.
 Failla JM, Amadio PC: Recognition and treatment of uncommon carpal fractures. Hand Clin 1988;4:469-476.
 Botte MJ, Gelberman RH: Fractures of the carpus, excluding the scaphoid. Hand Clin 1987;3:149-161.

34. A 32-year-old powerlifter who was performing a dead lift 3 days ago noted a sharp
pain in the front of his dominant right arm just after beginning to lower the weight.
He now reports pain in the anterior aspect of the arm that worsens when he opens a
Copyright©2001 by American Academy of Orthopaedic Surgeons™. All rights reserved.
56  American Academy of Orthopaedic Surgeons

door. Examination reveals moderate ecchymosis and swelling of the forearm and
tenderness in the antecubital fossa. The MRI scans are shown in Figures 15a and
15b. If the injury is left unrepaired, the greatest functional deficit will most likely
be the loss of

1- elbow extension motion.


2- elbow flexion strength.
3- forearm supination motion.
4- forearm pronation strength.
5- forearm supination strength.

PREFERRED RESPONSE: 5

DISCUSSION: A complete tear of the distal biceps brachii most often occurs from a large, rapid
eccentric elbow extension load. A pop or tearing sensation usually occurs, and a palpable defect
in the antecubital fossa is often present on examination. The treatment of choice is a direct
primary repair by a two-incision technique. If left unrepaired, the most disabling consequence is
the loss of forearm supination strength. It is unlikely that significant elbow or forearm motion
will be lost if the rupture is left unrepaired and early motion exercises are initiated. Elbow
flexion strength tends to return with time, but the loss of forearm supination strength remains
problematic.
 D'Alessandro DF, Shields CL Jr, Tibone JE, Chandler RW: Repair of distal biceps tendon ruptures in athletes. Am J Sports
Med 1993;21:114-119.
 Agins HJ, Chess JL, Hoekstra DV, Teitge RA: Rupture of the distal insertion of the biceps brachii tendon. Clin Orthop
1988;234:34-38.

35. Figure 16 shows the lateral radiograph of a patient who is scheduled to undergo an
anterior cruciate ligament (ACL) reconstruction. If the graft is tensioned at 20
degrees of flexion and the femoral tunnel is created by passing a reamer over the
guide wire marked "A," the resulting ligament reconstruction will excessively
American Academy of Orthopaedic Surgeons
2001 Sports Medicine Self-Assessment Examination

1- tighten as the knee extends past 10 degrees of flexion.


2- tighten as the knee flexes past 90 degrees.
3- loosen as the knee extends past 10 degrees of flexion.
4- loosen as the knee flexes past 30 degrees.
5- loosen as the knee flexes past 90 degrees.

PREFERRED RESPONSE: 2

DISCUSSION: If the femoral tunnel is created using guide wire A, it will be too far anterior in
the intercondylar notch. The distance between a central tibial insertion for the ACL and an
anterior femoral tunnel will progressively increase as the knee is flexed. Therefore, if the graft is
tensioned near extension, the ligament will excessively tighten as the knee flexes past 90
degrees. This will result in restricted knee flexion or failure of the graft as full flexion is gained.
There will be little effect on the ligament as it extends from 20 degrees to 0 degrees of flexion.
If the graft is tensioned in significant flexion (greater than 60 degrees), it will be excessively
loose as the knee fully extends
 Daniel DM, Fritschy D: Anterior cruciate ligament injuries, in DeLee JC, Drez D Jr (eds): Orthopaedic Sports Medicine:
Principles and Practice. Philadelphia, PA, WB Saunders, 1994, pp 1313-1360.
 Larson RL, Tailon M: Anterior cruciate ligament insufficiency: Principles of treatment. J Am Acad Orthop Surg 1994;2:26-
35.

36. Which of the following nerves is most commonly injured during revision surgery
following a Bristow procedure?
1- Dorsal scapular
2- Suprascapular
3- Axillary
4- Musculocutaneous
5- Ulnar
6-
PREFERRED RESPONSE: 4

DISCUSSION: Because of the previously transferred bone block of coracoid and short arm
flexors, the musculocutaneous nerve often scars along the anteroinferior glenohumeral capsule.
Mobilization of this tissue places the nerve at greatest risk. The axillary nerve is also potentially
at risk, but this is nonspecific to prior surgery, particularly the Bristow procedure.
 Norris TR: Complications following anterior instability repairs, in Bigliani LU (ed): Complications of Shoulder Surgery.
Baltimore, MD, Williams and Wilkins, 1993, pp 98-116.
 Flatow EL, Bigliani LU, April EW: An anatomic study of the musculocutaneous nerve and its relationship to the coracoid
process. Clin Orthop 1989;244:166-171.

Copyright©2001 by American Academy of Orthopaedic Surgeons™. All rights reserved.


58  American Academy of Orthopaedic Surgeons

37. A 17-year-old high school soccer player sustains an anterior cruciate ligament
(ACL) tear at the beginning of the season. An MRI scan confirms a complete ACL
tear with no meniscal injuries. The patient plans an early return to play and would
like to avoid surgery. Therefore, the patient and family should be advised that
nonsurgical management consisting of rehabilitative exercises and the use of a
functional knee brace will most likely result in
1- recurrent buckling with a probable meniscal tear.
2- limitation of motion with a delay in recovery.
3- a full return to activity with no limitations.
4- an improvement in overall performance.
5- an uneventful completion of the soccer season.

PREFERRED RESPONSE: 1

DISCUSSION: While there are athletes who can function at a full level with an ACL tear, they
are in the minority. As yet, there is no reliable way to predict the patients who will be able to
compensate for the loss of the ACL. Studies have confirmed the risk of recurrent instability and
meniscal injury in athletes with an ACL-deficient knee who participate in cutting sports. One
study showed that only 12 of 43 patients who attempted rehabilitation and bracing were able to
return successfully for the season. Another study showed that 17 of 31 athletes who were able to
return to their sport sustained 23 meniscal tears because of recurrent instability.
 Shelton WR, Barrett GR, Dukes A: Early season anterior cruciate ligament tears: A treatment dilemma. Am J Sports Med
1997;25:656-658.
 Snyder-Mackler L, Fitzgerald GK, Bartolozzi AR III, Ciccotti MG: The relationship between passive joint laxity and
functional outcome after anterior cruciate ligament injury. Am J Sports Med 1997;25:191-195.

38. A patient underwent anterior stabilization of the shoulder 6 months ago, and
examination now reveals lack of external rotation beyond 0 degrees. The patient
has a normal apprehension sign and normal strength, and the radiographs are
normal. Based on these findings, the patient is at greater risk for the development
of
1- recurring instability.
2- osteoarthritis.
3- osteonecrosis.
4- tear of the rotator cuff.
5- internal impingement.

PREFERRED RESPONSE: 2

DISCUSSION: Because the patient's shoulders are overtensioned anteriorly, premature


osteoarthritis may develop. This may create obligate translation posteriorly and increase the
interarticular pressure of the humeral head against the glenoid. Patients should achieve 20
degrees to 30 degrees of external rotation with the elbow at the side. Late degenerative arthritis
following a Putti-Platt procedure is associated with significant restriction of external rotation.
This patient's shoulder has a reduced risk of anterior instability, rotator cuff tear, and internal
impingement because of the limitation of motion.
 Hawkins RJ, Angelo RL: Glenohumeral osteoarthritis: A late complication of the Putti-Platt repair. J Bone Joint Surg Am
1990;72:1193-1197.
American Academy of Orthopaedic Surgeons
2001 Sports Medicine Self-Assessment Examination

 Norris TR: Complications following anterior instability repairs, in Bigliani LU (ed): Complications of Shoulder Surgery.
Baltimore, MD, Williams and Wilkins, 1993, pp 98-116.

39. A 13-year-old girl who competes in gymnastics reports the insidious onset of lateral
left elbow pain over the past 6 months. She also notes occasional catching episodes
in the elbow; however, she denies any history of trauma. Examination reveals
tenderness over the lateral epicondyle and extensor muscle origin. The elbow is
stable and has full flexion, but lacks 10 degrees of full extension. An AP plain
radiograph and an MRI scan are shown in Figures 17a and 17b. Management of
the elbow should

1- open excision of the radial head.


2- a cortisone injection into the
extensor muscle origin.
3- a tennis elbow release.
4- arthroscopic removal of loose
bodies and microfracture of the
crater.
5- rest, physical therapy, pulsed
electromagnetic therapy, and no
further gymnastic activities.

PREFERRED RESPONSE: 4

DISCUSSION: The radiograph and MRI scan show osteochondritis dissecans of the capitellum,
and the patient's history suggests a loose body. The treatment of choice is arthroscopic removal
of the loose body and microfracture of the crater. Excision of the radial head, a cortisone
injection, or tennis elbow release does not treat the pathology in the capitellum. Nonsurgical
treatment would not relieve the mechanical symptoms of the loose body or promote healing in
the crater.
 Baumgarten TE, Andrews JR, Satterwhite YE: The arthroscopic classification and treatment of osteochondritis dissecans of
the capitellum. Am J Sports Med 1998;26:520-530.
 Jackson DW, Silvino N, Reiman P: Osteochondritis in the female gymnast's elbow. Arthroscopy 1989;5:129-136.
 Ruch DS, Cory JW, Poehling GG: The arthroscopic management of osteochondritis dissecans of the adolescent elbow.
Arthroscopy 1998;14:797-803.

40. A 25-year-old man injures his shoulder while skiing. Examination reveals increased
passive external rotation, pain in the cocked position, and a positive lift-off test.
What is the most likely diagnosis?
1- Ruptured biceps tendon
2- Subscapularis tear
3- Anterior subluxation
4- Internal impingement syndrome
5- Locked posterior dislocation
Copyright©2001 by American Academy of Orthopaedic Surgeons™. All rights reserved.
60  American Academy of Orthopaedic Surgeons

PREFERRED RESPONSE: 2

DISCUSSION: A positive lift-off test and increased passive external rotation are diagnostic of a
subscapularis tear or detachment. Although a similar injury could produce anterior instability,
this will test the integrity of the subscapularis. A locked dislocation has limited passive
movement. A ruptured biceps tendon will most likely produce ecchymosis and findings similar
to supraspinatus trauma. Internal impingement is not associated with subscapularis weakness.
 Gerber C, Krushell RJ: Isolated rupture of the tendon of the subscapularis muscle: Clinical features in 16 cases. J Bone
Joint Surg Br 1991;73:389-394.
 Hawkins RJ, Bokor DJ: Clinical evaluation of the shoulder, in Rockwood CA, Matsen FA III (eds): The Shoulder.
Philadelphia, PA, WB Saunders, 1990, pp 149-177.

41. A college basketball player is struck in the eye by a player's hand while driving to
the basket. Fluorescein evaluation reveals the injury shown in Figure 18.
Management should consist of

1- administration of ophthalmic
corticosteroids and antibiotics with
application of an eye patch.
2- evaluation of intact visual fields and
pupillary responses prior to a return
to play.
3- consultation with an ophthalmologist
prior to emergent repair of the
damaged structure.
4- measurement of ocular pressure and fundoscopic examination in a properly lit
examination room.
5- strict bed rest with the head elevated, minimizing head motion during the healing
process.

PREFERRED RESPONSE: 1

DISCUSSION: The athlete has a corneal abrasion. Fluorescein staining identifies the break in
the epithelium when examined with ultraviolet light. Topical antibiotics are used as prophylaxis
against secondary bacterial infection, and the patch, applied with the lid closed, is used for
comfort and to promote epithelial healing. The accompanying symptoms, including pain,
tearing, and photophobia, are usually too intense to allow a return to play. Surgery is reserved
for a corneal laceration with associated loss of the anterior chamber. While a proper fundoscopic
examination may be a consideration, increased intraocular pressure is not typically associated
with this injury. Traumatic hemorrhage in the anterior chamber (hyphema) necessitates strict
bed rest during the early phases of healing; examination will most likely reveal the red fluid level
of blood settling inferiorly in the anterior chamber. It is often associated with increased
intraocular pressure.
 Brucker AJ, Kozart DM, Nichols CW, et al: Diagnosis and management of injuries to the eye and orbit, in Torg JS (ed):
Athletic Injuries to the Head, Neck, and Face. St Louis, MO, Mosby-Year Book, 1991, pp 650-670.
 Zagelbaum BM: Treating corneal abrasions and lacerations. Phys Sports Med 1997;25:38-44.
American Academy of Orthopaedic Surgeons
2001 Sports Medicine Self-Assessment Examination

42. In patient selection for meniscal allograft transplantation, which of the following
variables has the greatest influence on outcome?
1- Grade of chondromalacia
2- Limb alignment
3- Patient age
4- Patient weight
5- Postoperative level of activity

PREFERRED RESPONSE: 1

DISCUSSION: Many clinical studies to date show that the extent of arthritis is the most common
variable that has the greatest influence on outcome. The success rate of allograft transplantation
is significantly diminished in patients who have grade IV chondromalacia of the knee or notable
flattening and general joint incongruity.
 Carter TR: Meniscal allograft transplantation. Sports Med Arthroscopy Rev 1999;7:51-63.
 Garrett JC: Meniscal transplantation: A review of 43 cases with two- to seven-year follow-up. Sports Med Arthroscopy Rev
1993;2:164-167.
 van Arkel ER, de Boer HH: Human meniscal transplantation: Preliminary results at 2- to 5-year follow-up. J Bone Joint
Surg Br 1995;77:589-595.

43. A 10-year-old boy sustained an injury to the left knee. The radiographic findings
shown in Figure 19 are most commonly associated with injury to which of the
following structures?

1- Anterior cruciate ligament (ACL)


2- Posterior cruciate ligament (PCL)
3- Patellar tendon
4- Lateral capsule
5- Pes anserinus

PREFERRED RESPONSE: 1

DISCUSSION: The radiograph shows a bony avulsion of the ACL attachment site on the tibial
spine in this skeletally immature patient. In this age group, injury often results in failure of the
bony attachment site rather than the substance of the ligament. Avulsion of the patellar tendon
insertion site can occur, but this structure is located at the apophysis of the tibial tubercle. The
attachment site of the PCL is much more posterior. In adults, bony avulsion is more commonly
associated with PCL injuries than with ACL injuries. When a small bony avulsion of the lateral
capsule from the lateral tibial plateau is seen on the AP view, this finding is considered
pathognomonic of an ACL injury (Segond sign) in adults. The area of the pes anserinus is
anterior and distal; avulsion would be unusual.
Copyright©2001 by American Academy of Orthopaedic Surgeons™. All rights reserved.
62  American Academy of Orthopaedic Surgeons

 Baxter MP, Wiley JJ: Fractures of the tibial spine in children: An evaluation of knee stability. J Bone Joint Surg Br
1988;70:228-230.
 Meyers MH, McKeever FM: Fracture of the intercondylar eminence of the tibia. J Bone Joint Surg Am 1970;52:1677-1684.
 DeLee JC: Ligamentous injury of the knee, in Stanitski CL, DeLee JC, Drez D Jr (eds): Pediatric and Adolescent Sports
Medicine. Philadelphia, PA, WB Saunders, 1994, pp 406-432.

44. What is the single most important nutritional factor affecting athletic performance?
1- Maximum precompetition carbohydrate stores
2- Adequate carbohydrate consumption during competition
3- Maintenance of adequate serum sodium
4- Maintenance of adequate serum potassium
5- Maintenance of adequate hydration

PREFERRED RESPONSE: 5

DISCUSSION: Maintenance of adequate hydration is the single most important factor affecting
athletic performance. While carbohydrate loading may be beneficial for some endurance
athletes, the consumption of carbohydrates during exercise does not appear to be beneficial for
athletes engaged in events that last less than 1 hour. In general, athletes consuming a balanced
diet do not need electrolyte supplementation.
 Maughan RJ, Noakes TD: Fluid replacement and exercise stress: A brief review of studies on fluid replacement and some
guidelines for the athlete. Sports Med 1991;12:16-31.
 Barr SI, Costill DL, Fink WJ: Fluid replacement during prolonged exercise: Effects of water, saline, or no fluid. Med Sci
Sports Exerc 1991;23:811-817.

45. A right-handed 20-year-old college baseball pitcher has had a 6-month history of
vague right elbow pain while pitching. Examination reveals full flexion of the elbow
and a loss of only a few degrees of full extension. The elbow is stable, but palpation
reveals tenderness over the olecranon. Plain radiographs are inconclusive. MRI
and CT scans are shown in Figures 20a and 20b. Management should consist of
American Academy of Orthopaedic Surgeons
2001 Sports Medicine Self-Assessment Examination

1- repair of a triceps tendon avulsion.


2- arthroscopy of the elbow for removal of loose bodies.
3- arthroscopic removal of a posteromedial olecranon osteophyte.
4- internal fixation of an olecranon stress fracture.
5- rest, rehabilitation, and resumption of pitching when the fracture is healed.

PREFERRED RESPONSE: 5

DISCUSSION: The patient has a stress fracture of the olecranon that occurs with repetitive
throwing motions. If the fracture is not displaced, the initial treatment of choice is rest and
rehabilitation to maintain elbow motion, followed by aggressive strengthening at 6 to 8 weeks.
A light throwing program generally can begin at 8 to 12 weeks. Complete recovery may require
3 to 6 months. If the fracture is displaced or if nonsurgical management fails, surgery is
indicated for internal fixation of the stress fracture.

 Azar FM, Wilk KE: Nonoperative treatment of the elbow in throwers. Oper Tech Sports Med 1996;4:91-99.
 Griffin LY (ed): Orthopaedic Knowledge Uupdate: Sports Medicine. Rosemont, IL, American Academy of Orthopaedic
Surgeons, 1994, pp 191-203.

46. What is the most common associated pathology in patients who have suprascapular
nerve entrapment secondary to ganglion cysts?
1- Glenohumeral arthritis
2- Fracture of the clavicle
3- Tear of the rotator cuff
4- Rupture of the long head of the biceps tendon
5- Superior labrum anterior and posterior (SLAP) lesion

PREFERRED RESPONSE: 5

DISCUSSION: It is well known that suprascapular nerve entrapment can be secondary to many
entities, and its association with ganglion cysts and SLAP lesions has been well documented.
Because of a superior labral tear, synovial fluid will leak out of the joint underneath the labrum,
causing the cyst and secondary compression of the nerve.
 Fehrman DA, Orwin JF, Jennings RM: Suprascapular nerve entrapment by ganglion cysts: A report of six cases with
arthroscopic findings and review of the literature. Arthroscopy 1995;11:727-734.
 Iannotti JP, Ramesey ML: Arthroscopic decompression of a ganglion cyst causing suprascapular nerve compression.
Arthroscopy 1996;12:739-745.
 Moore TP, Fritts HM, Quick DC, Buss DD: Suprascapular nerve entrapment caused by supraglenoid cyst compression. J
Shoulder Elbow Surg 1997;6:455-462.

47. A 27-year-old runner training for his first marathon reports lateral knee pain after
an unusually long training run. He states that the most significant pain occurs while
running downhill. Examination of the patient while he is laying on the unaffected
side reveals increased pain when manual pressure is applied to the lateral femoral

Copyright©2001 by American Academy of Orthopaedic Surgeons™. All rights reserved.


64  American Academy of Orthopaedic Surgeons

epicondylar area during knee range of motion of 30 degrees to 45 degrees. What is


the most likely diagnosis?
1- Popliteal tendinitis
2- Iliotibial band friction syndrome
3- Excessive lateral pressure syndrome
4- Lateral meniscal tear
5- Stress fracture

PREFERRED RESPONSE: 2

DISCUSSION: Iliotibial band friction syndrome is one of the most common causes of lateral
knee pain in runners. It is caused by increased friction between the iliotibial band and the lateral
femoral condyle because of increased tension on the lateral structures. It may be caused by a
prominence of the lateral epicondyle or a malalignment of the lower extremity in the runner,
including genu varum, tibia vara, heel varus and forefoot supination, or compensating pronation.
These structural characteristics can couple with relative muscle imbalance and lead to an altered
running gait, enhancing friction between the lateral femoral condyle and the iliotibial band.
Management is usually nonsurgical, including stretching of the iliotibial band and strengthening
of the hip abductor muscles, with occasional use of cortisone injections or iontophoresis.
 Noble CA: The treatment of iliotibial band friction syndrome. Br J Sports Med 1979;13:51-54.
 James SL: Running injuries to the knee. J Am Acad Orthop Surg 1995;3:309-318.
 James SL, Jones DV: Biomechanical aspects of distance running, in Cavanagh PR (ed): Biomechanics of Distance Running.
Champaign, IL, Human Kinetic Books, 1990, pp 249-269.

48. A 30-year-old woman who runs approximately 30 miles a week has had right hip
and groin pain for the past 3 weeks. Examination reveals an antalgic gait, limited
motion of the right hip, and pain, especially with internal and external rotation.
Plain radiographs are normal, and an MRI scan is shown in Figure 21.
Management should consist of

1- immediate internal fixation of the right


femoral neck stress fracture.
2- non-weight-bearing crutch ambulation
until symptoms resolve, followed by a
gradual resumption of activities.
3- ultrasound therapy to promote fracture
healing.
4- a metabolic work-up.
5- a bone scan to look for other stress
fractures.

PREFERRED RESPONSE: 2

DISCUSSION: A stress fracture of the hip is a relatively common problem in endurance sports.
These fractures are classified as compression-side, tension-side, and displaced femoral neck
American Academy of Orthopaedic Surgeons
2001 Sports Medicine Self-Assessment Examination

fractures. The MRI scan shows a compression-side stress fracture. Compression-side fractures
usually occur in the inferior or calcar area of the proximal femur, and non-weight-bearing crutch
ambulation for 6 to 7 weeks will most likely result in healing. Once the patient is walking
without pain or a limp, activities can be slowly increased. Because tension-side fractures have a
high risk of displacement, treatment should consist of immediate internal fixation.

 Griffin LY (ed): Orthopaedic Knowledge Update: Sports Medicine. Rosemont, IL, American Academy of Orthopaedic
Surgeons, 1994, pp 239-253.
 Fullerton LR Jr, Snowdy HA: Femoral neck stress fractures. Am J Sports Med 1988;16:365-377.

49. Which of the following primary prognostic factors best predicts the outcome of the
knee lesion shown in Figure 22?

1- Location
2- Size
3- Knee stability
4- Patient age
5- Degree of pain

PREFERRED RESPONSE: 4

DISCUSSION: The patient has osteochondritis dissecans. While location, size, and knee
stability are all relevant to the overall prognosis, studies have shown that younger patients with
open growth plates have a better prognosis of healing when compared with patients who have
closed growth plates. The degree of pain is also relevant to treatment, but it is subjective rather
than objective and is not as reliable of a prognostic indicator as age.

 Stanitski CL: Osteochondritis dissecans of the knee, in Stanitski CL, DeLee JC, Drez D Jr (eds): Pediatric and Adolescent
Sports Medicine. Philadelphia, PA, WB Saunders, 1994, vol 3, pp 387-405.
 Cahill B: Treatment of juvenile osteochondritis dissecans and osteochondritis dissecans of the knee. Clin Sports Med
1985;4:367-384.
 Linden B: Osteochondritis dissecans of the femoral condyles: A long-term follow-up study. J Bone Joint Surg Am
1977;59:769-776.

50. Figures 23a and 23b show the AP and lateral radiographs of the elbow of a 30-year-
old professional pitcher. The pathology shown in these studies is most consistent
with which of the following conditions?

Copyright©2001 by American Academy of Orthopaedic Surgeons™. All rights reserved.


66  American Academy of Orthopaedic Surgeons

1- Insertional triceps tendinitis


2- Valgus extension overload
3- Medial epicondylitis
4- Stress fracture of the olecranon
5- Chronic olecranon bursitis

PREFERRED RESPONSE: 2

DISCUSSION: The radiographs show the osteophytic build-up of the posteromedial corner of
the elbow that occurs with valgus extension overload in the pitching elbow. This is the result of
excessive valgus forces during the acceleration and deceleration phases of throwing. These
forces, coupled with medial elbow stresses, cause a wedging of the olecranon into the medial
wall of the olecranon fossa. Valgus instability of the elbow may further stimulate osteophyte
formation. Repetitive impact of a spur within the olecranon fossa may cause fragmentation and
eventual formation of loose bodies.

 Azar FM, Wilk KE: Nonoperative treatment of the elbow in throwers. Oper Tech Sports Med 1996;4:91-99.
 Field LD, Savoie FJ: Common elbow injuries in sport. Sports Med 1988;26:193-205.
 Wilson FD, Andrews JR, Blackburn TA, et al: Valgus extension overload in the pitching elbow. Am J Sports
Med 1983;11:83-88.

51. Figure 24 shows the radiograph of a 10-year-old boy who sustained a valgus injury
to the knee. Examination reveals grade III medial laxity. Initial management
should consist of

1- an MRI scan.
2- stress radiographs of the knee.
3- activities as tolerated.
4- a hinged range-of-motion brace.
5- a knee immobilizer.

PREFERRED RESPONSE: 2

DISCUSSION: Based on the mechanism of injury and findings of medial laxity, the most likely
diagnosis is injury to either the growth plate or the medial collateral ligament. With the open
physeal plate, this area of injury is presumed present until proven otherwise; therefore, stress
radiographs should be obtained before implementing any treatment or ordering more extensive
and expensive tests.
 DeLee JC: Ligamentous injury of the knee, in Stanitski CL, DeLee JC, Drez D Jr (eds): Pediatric and Adolescent Sports
Medicine. Philadelphia, PA, WB Saunders, 1994, vol 3, pp 406-432.
American Academy of Orthopaedic Surgeons
2001 Sports Medicine Self-Assessment Examination

 Clanton TO, DeLee JC, Sanders B, Neidre A: Knee ligament injuries in children. J Bone Joint Surg Am 1979;61:1195-
1201.
 Torg JS, Pavlov H, Morris VB: Salter-Harris type III fracture of the medial femoral condyle occurring in the adolescent
athlete. J Bone Joint Surg Am 1981;63:586-591.

52. A right-handed 14-year-old pitcher has had a 3-month history of shoulder pain
while pitching. Examination reveals full range of motion, a mildly positive
impingement sign, pain with rotational movement, and no instability. Plain AP
radiographs of both shoulders are shown in Figures 25a and 25b. Management
should consist of

1- referral to a pitching coach to


improve throwing mechanics.
2- a weight-training program
that concentrates on rotator
cuff strengthening.
3- rest until symptoms have
resolved, followed by a
gradual return to pitching.
4- a metabolic work-up.
5- cessation of pitching until the
physis is closed.

PREFERRED RESPONSE: 3

DISCUSSION: The patient has the classic signs of Little Leaguer's shoulder, with findings that
include pain localized to the proximal humerus during the act of throwing and radiographic
evidence of widening of the proximal humeral physis. Examination usually reveals tenderness to
palpation over the proximal humerus, but the presence of any swelling, weakness, atrophy, or
loss of motion is unlikely. The treatment of choice is rest from throwing for at least 3 months,
followed by a gradual return to pitching once the shoulder is asymptomatic.
 Carson WG Jr, Gasser SI: Little Leaguer's shoulder: A report of 23 cases. Am J Sports Med 1998;26:575-580.
 Barnett LS: Little League shoulder syndrome: Proximal humeral epiphyseolysis in adolescent baseball pitchers. A case
report. J Bone Joint Surg Am 1985;67:495-496.

53. A 38-year-old man sustains a complete avulsion with retraction of the ischial
attachment of the hamstring muscles in a fall while water skiing. He indicates that
he is an aggressive athlete who participates regularly in multiple running and
cutting-type sports, and he strongly desires to continue his athletic competition.
Management should consist of
1- ultrasound, iontophoresis, and stretching, with an early return to sports.
2- a local corticosteroid injection and strengthening, with a delayed return to sports.
3- immobilization and rehabilitation, with a delayed return to sports.
Copyright©2001 by American Academy of Orthopaedic Surgeons™. All rights reserved.
68  American Academy of Orthopaedic Surgeons

4- early surgical repair, prolonged rehabilitation, and a return to sports.


5- rehabilitation, with delayed surgical repair if the patient is unable to return to
sports.

PREFERRED RESPONSE: 4

DISCUSSION: Several studies have identified a complete proximal avulsion of the hamstring
muscles as an injury that leads to significant long-term disability, with a high percentage of
athletes who must permanently restrict their activities following nonsurgical management. Early
surgical repair and prolonged rehabilitation have yielded consistently better results than
nonsurgical management.
 Orava S, Kujala UM: Rupture of the ischial origin of the hamstring muscles. Am J Sports Med 1995;23:702-705.
 Clanton TO, Coupe KJ: Hamstring strains in the athlete: Diagnosis and treatment. J Am Acad Orthop Surg 1998;6:237-248.

54. What mechanism contributes to strength gains during conditioning of the


preadolescent athlete?
1- Enhanced neurogenic adaptations
2- Advanced myogenic adaptations
3- Increased contractile proteins
4- Increased short-term energy sources
5- Thickening of the connective tissue

PREFERRED RESPONSE: 1

DISCUSSION: Prepubescent athletes gain strength through neurogenic adaptations, including


recruitment of motor units, reduced inhibition, and learned motor skills. Myogenic adaptations
(muscle hypertrophy) occur after puberty and include increased contractile proteins, thickening
of the connective tissue, and increased short-term energy sources such as creatine phosphate
 Grana WA: Strength training, in Stanitski CL, DeLee JC, Drez D Jr (eds): Pediatric and Adolescent Sports Medicine.
Philadelphia, PA, WB Saunders, 1994, pp 520-526.
 Micheli LJ: Strength training, in Sullivan JA, Grana WA (eds): The Pediatric Athlete. Park Ridge, IL, American Academy
of Orthopaedic Surgeons, 1990, pp 17-20.

55. Following an episode of transient quadriplegia in contact sports, an athlete's return


to play is absolutely contraindicated when
1- the spinal canal to vertebral body ratio (Torg ratio) is less than or equal to 0.8.
2- electromyelographic studies are abnormal.
3- MRI scans or contrast-enhanced CT scans show severe spinal stenosis.
4- unilateral burning pain persists.
5- the episode of quadriplegia lasts 5 minutes.

PREFERRED RESPONSE: 3
American Academy of Orthopaedic Surgeons
2001 Sports Medicine Self-Assessment Examination

DISCUSSION: Return to play decisions after traumatic spinal or spinal cord injury are not
always clear-cut and often must be made on a patient-by-patient basis. The Torg ratio has been
found to have low sensitivity in patients with large vertebral bodies. Abnormal
electromyographic studies can persist in the face of normal function and do not define spinal
injury. Duration of quadriplegia is not related to anatomic pathology. Findings on MRI scans or
contrast-enhanced CT scans consistent with stenosis include lack of a significant cerebrospinal
fluid signal around the cord, bony or ligament hypertrophy, or disk encroachment. Based on
these findings, return to play should be avoided.
 Cantu RC, Bailes JE, Wilberger JE Jr: Guidelines for return to contact or collision sport after a cervical spine injury. Clin
Sports Med 1998;17:137-146.
 Herzog RJ, Wiens JJ, Dillingham MF, Sontag MJ: Normal cervical spine morphometry and cervical stenosis in
asymptomatic professional football players: Plain film radiography, multiplanar computer tomography, and magnetic
resonance imaging. Spine 1991;16:178-186.
 Bailes JE, Hadley MN, Quigley MR, Sonntag VK, Cerullo LJ: Management of athletic injuries of the cervical spine and
spinal cord. Neurosurgery 1991;29:491-497.

56. A 16-year-old snowboarder has significant pain and is still unable to bear weight
after sustaining a lateral ankle injury in a fall 1 week ago. Examination reveals
swelling and tenderness in the sinus tarsi. AP, lateral, and mortise radiographs of
the ankle are unremarkable. Management should consist of
1- an elastic bandage, cold packs, and weight bearing as tolerated.
2- non-weight-bearing and a CT scan of the talus.
3- cast immobilization for 10 days, followed by progressive rehabilitation.
4- cast immobilization for 6 weeks, followed by progressive rehabilitation.
5- stirrup splinting, cold packs, and aggressive rehabilitation.

PREFERRED RESPONSE: 2

DISCUSSION: Because there is a significant possibility that the patient may have a fracture of
the lateral process of the talus, there is some disagreement as to the best radiographic study to
identify this injury. A CT scan is an appropriate diagnostic tool to visualize the fracture and
identify any displacement. Displaced lateral process fractures are best treated surgically
 Kirkpatrick DP, Hunter RE, Janes PC, Mastrangelo J, Nicholas RA: The snowboarder's foot and ankle. Am J Sports Med
1998;26:271-277.
 Ebraheim NA, Skie MC, Podeszwa DA, Jackson WT: Evaluation of process fractures of the talus using computed
tomography. J Orthop Trauma 1994;8:332-337.

57. A 24-year-old man who plays golf noted the immediate onset of pain on the ulnar
side of his hand and has been unable to swing a club for the past 6 weeks after
striking a tree root with his club during his golf swing. Examination reveals full
motion of the wrist, diminished grip strength, and tenderness over the hypothenar
region. A CT scan of the hand and wrist is shown in Figure 26. Management
should consist of

Copyright©2001 by American Academy of Orthopaedic Surgeons™. All rights reserved.


70  American Academy of Orthopaedic Surgeons

1- immobilization of the wrist


until the fracture heals.
2- excision of the hook of the
hamate.
3- internal fixation of the
fractured hook of the hamate.
4- ultrasound therapy to promote
fracture healing.
5- limited intercarpal arthrodesis.

PREFERRED RESPONSE: 1

DISCUSSION: Fractures of the hook of the hamate frequently are not identified in the acute
phase. Because the fracture can be difficult to see on plain radiographs, the lack of findings can
lead to a painful nonunion. A carpal tunnel view may show the fracture, but a CT scan will best
detect the injury. Immobilization is the treatment of choice and will result in union in most
patients unless the diagnosis is delayed. However, excision of the fragment may be necessary
for patients who have nonunion, persistent pain, or ulnar nerve palsy.

 Carroll RE, Lakin JF: Fracture of the hook of the hamate: Acute treatment. J Trauma 1993;34:803-805.
 Whalen JL, Bishop AT, Linscheid RL: Nonoperative treatment of acute hamate hook fractures. J Hand Surg Am
1992;17:507-511.

58. An 18-year-old football player sustains a contact injury to his right lower leg, and
radiographs show a closed transverse fracture of the middle third of the tibia.
Based on the clinical examination, a compartment syndrome is suspected. When
measuring compartment pressures, the highest tissue pressure is recorded how
many centimeters proximal or distal to the fracture site?
1- 0 cm to 5 cm
2- 5 cm to 10 cm
3- 10 cm to 15 cm
4- 15 cm to 20 cm
5- Greater than 20 cm

PREFERRED RESPONSE: 1

DISCUSSION: Measurements of compartment pressures in patients with tibial fractures and


compartment syndrome reveal that the highest tissue pressures are recorded at the level of the
fracture or within 5 cm of the fracture. Tissue pressures show a statistically significant decrease
when they are recorded at increasing distances proximal and distal to the site of the highest
pressure recorded. To reliably determine the location of the highest tissue pressure in patients
with tibial fractures, measurements should be obtained, at a minimum, in both the anterior and
deep posterior compartments at the level of the fracture, as well as at locations proximal and
American Academy of Orthopaedic Surgeons
2001 Sports Medicine Self-Assessment Examination

distal. The highest tissue pressure recorded should serve as a basis for determining the need for
fasciotomy.
 Heckman MM, Whitesides TE Jr, Grewe SR, Rooks MD: Compartment pressure in association with closed tibial fractures:
The relationship between tissue pressure, compartment, and the distance from the site of the fracture. J Bone Joint Surg Am
1994;76:1285-1292.
 Whitesides TE Jr, Heckman MM: Acute compartment syndrome: Update on diagnosis and treatment. J Am Acad Orthop
Surg 1996;4:209-218.

59. A 50-year-old patient who plays tennis sustained the deformity shown in Figure 27
following a high volley. Further diagnostic work-up should include

1- an electromyogram (EMG) of
the upper extremity.
2- an ultrasound of the short
head of the biceps.
3- an MRI scan of the rotator
cuff.
4- a CT scan with contrast of the
anterior labrum.
5- a subclavian venogram.

PREFERRED RESPONSE: 3

DISCUSSION: The patient has a rupture of the long head of the biceps; however, patients older
than age 45 years are at greater risk of having an associated rotator cuff tear. An MRI scan
should be ordered to avoid missing concomitant rotator cuff pathology. While patients may
report pain radiating down the arm at the time of the tendon rupture, an EMG is not indicated.
The short head of the biceps is intact and needs no further work-up, even though the muscle
descends in most cases. The anterior labrum can be injured but is not associated with this
deformity
 Neer CS II, Bigliani LU, Hawkins RJ: Rupture of the long head of the biceps related to the subacromial impingement.
Orthop Trans 1977;1:114.
 Hawkins RJ, Murnaghan JP: The shoulder, in Gruess RL, Ronnie WRJ (eds): Adult Orthopaedics. New York, NY,
Churchill Livingstone, 1984, pp 945-1054.

60. A 16-year-old ice hockey player is struck on the chest by the puck. He skates a few
strides and then collapses. What is the most likely diagnosis?
1- Acute aortic dissection
2- Pulmonary contusion
3- Commotio cordis
4- Acute cardiac tamponade
5- Splenic rupture

PREFERRED RESPONSE: 3
Copyright©2001 by American Academy of Orthopaedic Surgeons™. All rights reserved.
72  American Academy of Orthopaedic Surgeons

DISCUSSION: Sudden cardiac arrest following a blow to the chest in young athletes has been
termed "commotio cordis." It is most common in Little League and other youth projectile sports
(eg, ice hockey, lacrosse). The cause, although not completely determined, is most likely an
arrhythmia related to the impact in a vulnerable time in the cardiac cycle. Resuscitation has
proven to be exceedingly difficult, resulting in a high mortality rate.
 Maron BJ, Strasburger JF, Kugler JD, Bell BM, Brodkey FD, Poliac LC: Survival following blunt chest impact-induced
cardiac arrest during sports activities in young athletes. Am J Cardiol 1997;79:840-841.
 Link MS, Maron BJ, Estes NAM III: Commotio cordis, in Estes NAM III, Salem DN, Wang PJ (eds): Sudden Cardiac
Death in the Athlete. Armonk, NY, Futura, 1998, pp 515-528.

61. A 24-year-old dancer sustains the injury shown in Figure 28. Management should
consist of

1- closed reduction and application of


a well-molded cast.
2- open reduction and percutaneous
pin fixation.
3- open reduction and internal
fixation with a mini fragment plate
and screws.
4- intramedullary screw fixation.
5- brief immobilization and
symptomatic treatment.

PREFERRED RESPONSE: 5

DISCUSSION: The patient has a moderately displaced distal diaphyseal fracture of the fifth
metatarsal, and the most appropriate treatment is brief immobilization and symptomatic
management. Attempts at closed reduction are unlikely to appreciably alter the position of the
fracture. Surgical techniques for either reduction of the fracture or fixation have not been shown
to result in improved functional outcomes.
 O'Malley MJ, Hamilton WG, Munyak J: Fractures of the distal shaft of the fifth metatarsal: "Dancer's Fracture." Am J
Sports Med 1996;24:240-243.
 DeLee JC: Fractures and dislocations of the foot, in Mann RA, Coughlin MJ (eds): Surgery of the Foot and Ankle, ed 6. St
Louis, MO, CV Mosby, 1993, pp 1465-1703
 Hamilton WG: Foot and ankle injuries in dancers, in Yokum L (ed): Sports Clinics of North America. Philadelphia, PA,
Williams and Wilkins, 1988.

62. A 22-year-old volleyball player has atrophy of the infraspinatus muscle. This deficit is
the result of entrapment of what nerve?
1- Axillary nerve in the posterolateral space
2- Dorsal scapular nerve at the medial border of the scapula
3- Suprascapular nerve in the scapular notch
American Academy of Orthopaedic Surgeons
2001 Sports Medicine Self-Assessment Examination

4- Suprascapular nerve in the spinoglenoid notch


5- Subscapular nerve at the rotator interval

PREFERRED RESPONSE: 4

DISCUSSION: Suprascapular deficits, as the result of repetitive forceful internal rotation with
overhead ball striking, occur in the spinoglenoid notch. Compression interferes with distal
suprascapular nerve innervation to the infraspinatus, while allowing the supraspinatus to function
normally. A scapular notch entrapment of this nerve would involve both the supraspinatus and
the infraspinatus. The axillary, dorsal scapular, and subscapular nerves do not affect the
infraspinatus.
 Ferretti A, Cerullo G, Russo G: Suprascapular neuropathy in volleyball players. J Bone Joint Surg Am 1987;69:260-263.
 Bigliani LU, Dalsey RM, McCann PD, April EW: An anatomical study of the suprascapular nerve. Arthroscopy
1990;6:301-305.

63. Figure 29 shows the radiograph of a 25-year-old woman who has had a 3-month
history of ankle pain after sustaining an inversion injury to the ankle. She reports
occasional catching, but no sense of instability. Examination reveals ligament
stability. Management should consist of

1- a non-weight-bearing short leg cast.


2- open reduction and internal fixation.
3- no weight bearing with motor exercises for 8
weeks.
4- debridement, curettage, and drilling.
5- an ankle brace or taping when participating in
athletic activity.

PREFERRED RESPONSE: 4

DISCUSSION: Osteochondral lesions of the talar dome can have a traumatic or nontraumatic
etiology. Most authors site a probable traumatic etiology for lateral lesions. Stage I and II
lesions, which are composed of compressed subchondral bone or a partial detached
osteochondral fragment, can be treated initially in a non-weight-bearing short leg cast for 6
weeks. Stage III medial lesions can also be treated in the same manner. If symptoms persist, the
treament of choice is debridement of the fracture, curettage of the lesion, and drilling of the
subchondral bone. This treatment also applies to lateral stage III and all stage IV lesions. If the
fragment is at least one third of the size of the talar dome, management should consist of open
reduction and internal fixation. In patients with more chronic lesions (4 to 6 months of persistent
pain), the threshold to proceed with surgery is lower, even in a stage II lesion.
 Lutter LD, Mizel MS, Pfeffer GB (eds): Orthopaedic Knowledge Update: Foot and Ankle. Rosemont, IL, American
Academy of Orthopaedic Surgeons, 1994, pp 205-226.
 Pettine KA, Morrey BF: Osteochondral fractures of the talus: A long-term follow-up. J Bone Joint Surg Br 1987;69:89-92.

Copyright©2001 by American Academy of Orthopaedic Surgeons™. All rights reserved.


74  American Academy of Orthopaedic Surgeons

64. A 19-year-old college cross-country runner is amenorrheic and has recurrent stress
fractures. Long-term management should consist of
1- cross training with swimming and cycling.
2- a complete cessation of running.
3- vitamin D and calcium supplements
4- increased caloric intake.
5- oral contraceptives, vitamin D, and calcium supplements.

PREFERRED RESPONSE: 5

DISCUSSION: The triad of menstrual dysfunction, disordered eating, and stress fracture is well
recognized in women who participate in endurance sports. The best treatment remains to be
determined, but at present, the combination of oral contraceptives to regulate menses, an
increased intake of calcium and vitamin D, as well as nutritional counseling, is the recommended
treatment for decreased bone mass related to exercise-induced amenorrhea.
 Nattiv A, Armsey TD Jr: Stress injury to bone in the female athlete. Clin Sports Med 1997;16:197-224.
 Drinkwater BL: Exercise and bones: Lessons learned from female athletes. Am J Sports Med 1996;24:S33-S35.

65. A 47-year-old male tennis player has pain in his nondominant shoulder that has
failed to respond to 4 months of nonsurgical management. Examination reveals
acromial tenderness and pain at the supraspinatus tendon insertion. He has a
positive impingement sign, pain on forward elevation, and minimal cuff weakness.
The MRI scans are shown in Figures 30a and 30b. To completely resolve his
symptoms, treatment should consist of

1- rigid open reduction and internal fixation of the os acromiale with autologous
bone graft.
2- arthroscopic repair of the rotator cuff and acromioplasty.
3- arthroscopic excision of the os acromiale.
4- arthroscopic decompression of the supraglenoid cyst.
American Academy of Orthopaedic Surgeons
2001 Sports Medicine Self-Assessment Examination

5- open distal clavicle excision (Mumford procedure).

PREFERRED RESPONSE: 1

DISCUSSION : The MRI scans show a mesoacromion with tendonopathy of the supraspinatus.
The history and physical findings indicate that the patient has a symptomatic os acromiale.
Simple excision of the unstable os acromiale has not yielded consistently good results.
Meticulous internal fixation using tension banding with cannulated screws and autologous bone
grafting has shown good results for this problem.
 Hutchinson MR, Veenstra MA: Arthroscopic decompression of shoulder impingement secondary to os acromiale.
Arthroscopy 1993;9:28-32.
 Warner JJ, Beim GM, Higgins L: The treatment of symptomatic os acromiale. J Bone Joint Surg Am 1998;80:1320-1326.

66. A 39-year-old competitive cyclist sustains an injury to her left hip in a fall.
Gadolinium arthrography, with an accompanying MRI scan, is shown in Figure 31.
A cleft, or defect, identified by the arrow, indicates a detachment of the

1- acetabular labrum.
2- zona orbicularis.
3- iliofemoral ligament.
4- acetabular pulvinar.
5- retinacular vessels.

PREFERRED RESPONSE: 1

DISCUSSION : The area indicated by the arrow represents gadolinium contrast extending into a
separation between the lateral labrum and its acetabular attachment. This can be a traumatic
detachment, but occasionally a cleft may be present as a normal variant of the labral
morphology. The capsular attachment of the iliofemoral ligament is peripheral to the labrum.
The pulvinar is the common name applied to the fat and overlying synovium contained within
the acetabular fossa above the ligamentum teres. The zona orbicularis is a circumferential
thickening of the capsule around the femoral neck, and the retinacular vessels travel within the
capsular synovium up the femoral neck to supply the femoral head.

 Petersilge CA, Haque MA, Petersilge WJ, Lewin JS, Lieberman JM, Buly R: Acetabular labral tears: Evaluation with MR
arthrography. Radiology 1996;200:231-235.
 Czerny C, Hofmann S, Neuhold A, et al: Lesions of the acetabular labrum: Accuracy of MR imaging and MR arthrography
in detection and staging. Radiology 1996;200:225-230.
 Byrd JWT: Indications and contraindications, in Byrd JWT (ed): Operative Hip Arthroscopy. New York, NY, Thieme,
1998, pp 7-24.

Copyright©2001 by American Academy of Orthopaedic Surgeons™. All rights reserved.


76  American Academy of Orthopaedic Surgeons

67. A cortisone injection in the subacromial space will most likely result in
1- elevated blood glucose levels in patients with diabetes.
2- increased instability in multidirectional patients.
3- accelerated rupture of the long head of the biceps.
4- accelerated osteoporosis of the tuberosity.
5- altered proprioception of the glenohumeral joint.

PREFERRED RESPONSE: 1

DISCUSSION : A cortisone injection in the subacromial space will most likely result in elevated
blood glucose levels in patients with type I diabetes mellitus. Patients should be warned of this
potential complication. Cortisone does not have an effect on instability or proprioception, and a
single injection would not affect osteoporosis. Repetitive injections or injection into the tendon
itself could accelerate rupture of the biceps tendon.
 Matsen FA III, Arntz CT: Subacromial impingement, in Rockwood CA, Matsen FA III (eds): The Shoulder. Philadelphia,
PA, WB Saunders, 1990, pp 623-646.
 Koehler BE, Urowitz MB, Killinger DW: The systemic effects of intra-articular corticosteroid. J Rheumatol 1974;1:117-
125.

68. A high school athlete reports the sudden onset of low back pain while performing a
dead lift. Examination reveals a lumbar paraspinal spasm and a positive straight
leg raising test. The deep tendon reflexes, motor strength, and sensation in the
lower extremeties are normal. The radiographs are normal. If symptoms persist
for more than a few weeks, management should consist of
1- an electromyogram and nerve conduction velocity studies.
2- an MRI scan.
3- a bone scan.
4- physical therapy.
5- bed rest.

PREFERRED RESPONSE: 2

DISCUSSION : In adolescents, a lumbar herniated disk is characterized by a paucity of clinical


findings; a positive straight leg raising test may be the only consistent positive finding. This may
result in a long period of nonsurgical management that fails to provide relief. Activities that
place a significant shear load on the lumbar spine, such as the dead lift, are associated with an
increased risk of central disk herniation. When an adolescent who lifts weights has a history of
low back pain that fails to respond to a short period of active rest, an MRI scan is the study of
choice to evaluate for a lumbar herniated disk.

 Epstein JA, Epstein NE, Marc J, Rosenthal AD, Lavine LS: Lumbar intervertebral disk herniation in teenage children:
Recognition and management of associated anomalies. Spine 1984;9:427-432.
 Hashimoto K, Fujita K, Kojimoto H, Shimomura Y: Lumbar disc herniation in children. J Pediatr Orthop 1990;10:394-396.
American Academy of Orthopaedic Surgeons
2001 Sports Medicine Self-Assessment Examination

69. A 22-year-old skier reports painful range of motion in the left thumb after falling
forward on his outstretched hand while holding his ski pole. Examination of the left
thumb reveals increased AP laxity and 45 degrees of valgus laxity at the
metacarpophalangeal (MCP) joint. Examination of the right thumb shows 25
degrees of valgus laxity at the MCP joint. Radiographs are normal. Management
should consist of
1- primary repair of the ulnar collateral ligament.
2- volar plate arthroplasty.
3- pinning of the MCP joint for 6 weeks.
4- a thumb spica cast.
5- a hand-based thumb spica splint.

PREFERRED RESPONSE: 1

DISCUSSION : The patient has a complete tear of the ulnar collateral ligament as defined by
MCP joint laxity of greater than 30 degrees (or 15 degrees greater laxity compared with the
opposite side). Primary repair is the treatment of choice because displacement of the ligament
superficial to the adductor aponeurosis (Stener lesion) must be corrected. Any volar plate injury
can be addressed during repair of the ulnar collateral ligament

 Heyman P: Injuries to the ulnar collateral ligament of the thumb metacarpophalangeal joint. J Am Acad Orthop Surg
1997;5:224-229.

70. Which of the following structures is most commonly involved in lateral


epicondylitis?
1- Anconeus
2- Extensor digitorum communis
3- Extensor carpi radialis longus
4- Extensor carpi radialis brevis
5- Extensor carpi ulnaris

PREFERRED RESPONSE: 4

DISCUSSION : The most common specific site of involvement is the origin of the extensor carpi
radialis brevis. It is usually caused by overuse activities, such as the eccentric overload exhibited
during a backhand in tennis. In most patients, the characteristic friable, grayish tissue described
as angiofibroblastic hyperplasia or hyaline degeneration is seen at the extensor carpi radialis
brevis origin.

 Nirschl RP: Elbow tendinosis/tennis elbow. Clin Sports Med 1992;11:851-870.


 Regan W, Wold LE, Coonrad R, Morrey BF: Microscopic histopathology of chronic refractory lateral epicondylitis. Am J
Sports Med 1992;20:746-749.

Copyright©2001 by American Academy of Orthopaedic Surgeons™. All rights reserved.


78  American Academy of Orthopaedic Surgeons

71. When comparing surgical and nonsurgical extremities in patients who underwent
anterior cruciate ligament (ACL) reconstruction using patellar tendon or
hamstrings autografts, isokinetic strength measurements obtained 6 months after
the surgery would most likely reveal
1- significant quadricep weakness in the pateller tendon compared with the
hamstring.
2- quadricep weakness in the hamstring compared with the pateller tendon.
3- significant weakness in the hamstring compared with the patellar tendon.
4- significant hamstring weakness in the pateller tendon compared with the
hamstring.
5- no significant difference between the hamstring and the pateller tendon.

PREFERRED RESPONSE: 5

DISCUSSION : Follow-up examination at 6 months revealed no statistically significant


differences in quadricep or hamstring strength when comparing surgical versus nonsurgical
extremities isokinetically. Therefore, the selection of autogenous hamstring or patellar tendon
for ACL reconstruction should not be based solely on the assumption of the graft tissue source
altering the recovery of quadricep and/or hamstring strength.
 Carter TR, Edinger S: Isokinetic evaluation of anterior cruciate ligament reconstruction: Hamstring versus patellar tendon.
Arthroscopy 1999;15:169-172
 Howell SM, Taylor MA: Brace-free rehabilitation, with early return to activity, for knees reconstructed with a double-
looped semitendinosus and gracilis graft. J Bone Joint Surg Am 1996;78:814-825.
 Shelbourne KD, Nitz P: Accelerated rehabilitation after anterior cruciate ligament reconstruction. Am J Sports Med
1990;18:292-299.

72. A quarterback sustains a rough tackle after which he appears confused, has a dazed
look on his face and an unsteady gait on standing. He denies loss of consciousness.
Reexamination within 10 minutes is normal, the patient is lucid, and he wants to
return to play. The coach and the player should be advised that he may
1- return to play immediately.
2- return to play in 1 week, if asymptomatic.
3- return to play in 1 month, if asymptomatic.
4- return only after a screening CT scan.
5- not return to play for the season.

PREFERRED RESPONSE: 1

DISCUSSION : The patient has a grade I (mild) concussion that can result in confusion and
disorientation, without loss of consciousness. This concussion syndrome is completely
reversible, with no long-term sequelae. Athletes who sustain a grade I concussion may return to
play after 15 minutes if there are no lingering symptoms, such as headache or vertigo. A grade II
concussion is characterized by loss of consciousness of less than 5 minutes. With this type of
injury, the athlete can return to play in 1 week, if asymptomatic. If a grade III (severe)
concussion is sustained, the athlete should avoid contact for a minimum of 1 month before
considering a return to competition. A grade III concussion is characterized by a loss of
American Academy of Orthopaedic Surgeons
2001 Sports Medicine Self-Assessment Examination

consciousness of greater than 5 minutes or posttraumatic amnesia of greater than 24 hours. A


CT scan is not indicated in a grade I injury. An athlete who sustains three grade I or grade II
concussions, or two grade III concussions may not return to play for the season.

 Torg JS, Gennarelli TA: Head and cervical spine injuries, in DeLee JC, Drez D Jr (eds): Orthopaedic Sports Medicine
Principles & Practice. Philadelphia, PA, WB Saunders, 1994, vol 1, pp 417-462.
 Cantu RC: Criteria for return to competition after closed head injury, in Torg JS (ed): Athletic Injuries to the Head, Neck,
and Face. St Louis, MO, Mosby, 1991.

73. The bone avulsion shown in Figure 32 has a high correlation with tearing of the

1- iliotibial band.
2- anterior cruciate ligament.
3- posterior cruciate ligament.
4- lateral collateral ligament.
5- biceps femoris tendon.

PREFERRED RESPONSE: 2

DISCUSSION : As described by Segond in 1987, an avulsion fracture of the lateral tibial plateau
is commonly referred to as a Segond fracture. Subsequent to 1987, several authors have also
found that the lateral capsular sign represents, but is not limited to, a disruption of the middle
third of the lateral capsule and a tear of the anterior cruciate ligament.

 Bach BR, Warren RF: Radiographic indicators of anterior cruciate ligament injury, in Feagin JA (ed): The Crucial
Ligaments. New York, NY, Churchill Livingston, 1988, pp 301-327.
 Segond P: Recherches cliniques et experimentales sur les epanchements sanguins du genou par entorse. Prog Med (Paris)
1987;7:297.
 Johnson LL: Lateral capsular ligament complex: Anatomical and surgical considerations. Am J Sports Med 1979;7:156-
160.

74. A 21-year-old college defensive lineman sustains a minimally displaced (less than 1
mm) midthird scaphoid fracture during the first game of the season. Management
should consist of:
1- cast immobilization and a return to play as symptoms allow.
2- cast immobilization and a return to play when union is achieved.
3- open reduction and internal fixation, followed by early range of motion with a
return to play when union is achieved.
4- open reduction and internal fixation, followed by a return to play with protective
casting.
5- symptomatic treatment, with definitive treatment at the end of the season.

Copyright©2001 by American Academy of Orthopaedic Surgeons™. All rights reserved.


80  American Academy of Orthopaedic Surgeons

PREFERRED RESPONSE: 1

DISCUSSION : The union rate for minimally displaced midthird scaphoid fractures is quite high
with cast immobilization while allowing a return to sports. Inadequate immobilization results in
a much higher nonunion rate. Early fixation and rehabilitation have been proposed for sports or
positions that are not amenable to cast immobilization. While immobilization of a nondisplaced
fracture results in an acceptably high union rate, there is no advantage to fixation in conjunction
with immobilization in the course of healing. With adequate immobilization and protection, play
restrictions until healing has occurred are unnecessary.

 Rettig AC, Kollias SC: Internal fixation of acute stable scaphoid fractures in the athlete. Am J Sports Med 1996;24:182-186
 Rettig AC, Weidenbener EJ, Gloyeske R: Alternative management in midthird scaphoid fractures in the athlete. Am J
Sports Med 1994;22:711-714.
 Riester JN, Baker BE, Mosher JF, Lowe D: A review of scaphoid fracture healing in competitive athletes. Am J Sports Med
1985;13:159-161.

75. A 16-year-old football player sustains a direct blow to the anterior aspect of his
flexed right knee. Examination reveals a contusion over the anterior tibial tubercle
and a small effusion. MRI scans are shown in Figures 33a through 33c. What is the
most likely diagnosis?

1- Partial tear of the patellar tendon


2- Osteochondral fracture of the femur
3- Anterior cruciate ligament (ACL) tear
4- Posterior cruciate ligament (PCL) tear
5- Patella fracture

PREFERRED RESPONSE: 4

DISCUSSION : The MRI scans show disruption of the fibers of the PCL. Patients sustaining an
isolated acute PCL injury can present with only minimal discomfort and have full range of
motion. When examination reveals a contusion over the tibial tubercle and discomfort with the
posterior drawer examination, with or without instability, a possible injury to the PCL should be
American Academy of Orthopaedic Surgeons
2001 Sports Medicine Self-Assessment Examination

considered. In acute injuries, the reported accuracy of MRI imaging for diagnosing PCL tears
ranges from 96% to 100%.
 Resnick D, Kang HS: Internal Derangement of Joints: Emphasis on MRI Imaging. Philadelphia, PA, WB Saunders, 1997,
pp 699-700.
 Harner CD, Hoher J: Evaluation and treatment of posterior cruciate ligament injuries. Am J Sports Med 1998;26:471-482.
 Fischer SP, Fox JM, Del Pizzo W, Friedman MJ, Snyder SJ, Ferkel RD: Accuracy of diagnoses from magnetic imaging of
the knee: A multi-center analysis of one thousand and fourteen patients. J Bone Joint Surg Am 1991;73:2-10.

76. A 15-year-old diver has had persistent, activity-related low back pain for the past 2
months. He denies any history of trauma. Examination reveals that the pain is
localized to the lumbosacral junction, and there are no radicular symptoms. The
pain is worse with back extension. Neurologic examination is normal, as are AP,
lateral, and oblique radiographs of the lumbosacral spine. Further evaluation
should include
1- flexion and extension radiographs of the lumbosacral spine.
2- diskography.
3- an MRI scan of the lumbosacral spine.
4- a bone scan with single proton emission computed tomography (SPECT).
5- a renal ultrasound.

PREFERRED RESPONSE: 4

DISCUSSION : Spondylolysis may develop as a stress fracture resulting from repetitive


hyperextension during athletic activities. In young people, the pars interarticularis is thin, the
neural arch has not yet reached maximum strength, and the intravertebral disk is less resistant to
shear. While clinical symptoms may lead to the suspicion of spondylolysis, radiographic
confirmation may be difficult in early cases. Plain radiographs may be negative initially, and the
plain MRI scan may not offer good visualization of the pars. A bone scan with SPECT is very
sensitive initially. CT scans with regular axial and reverse-gantry angled cuts may help
determine the type of fracture and the course of treatment.
 Congeni J, McCulloch J, Swanson K: Lumbar spondylolysis: A study of natural progression in athletes. Am J Sports Med
1997;25:248-253.
 Harvey CJ, Richenberg JL, Saifuddin A, Wolman RL: The radiological investigation of lumbar spondylolysis. Clin Radiol
1998;53:723-728.

77. A 23-year-old college basketball player reports persistent lateral ankle pain after
sustaining an inversion injury 6 months ago. Examination reveals pain over the
anterolateral ankle, absence of swelling, and no clinical instability. Management
consisting of vigorous physical therapy fails to provide relief, and a intra-articular
corticosteroid injection provides only temporary relief. Radiographs obtained at
the time of injury and subsequent AP and varus stress views are normal. A recent
MRI scan fails to show any abnormalities. Management should now include
1- cast immobilization.
2- arthroscopy.
Copyright©2001 by American Academy of Orthopaedic Surgeons™. All rights reserved.
82  American Academy of Orthopaedic Surgeons

3- continued physical therapy.


4- a repeat corticosteroid injection.
5- a short course of oral steroids.

PREFERRED RESPONSE: 2

DISCUSSION : Because the patient has failed to respond to appropriate nonsurgical management
and imaging studies are normal, the use of arthroscopy not only aids in the diagnosis of chronic
ankle pain, but is also helpful in its treatment. In patients with this condition, typical findings
include synovitis in the lateral gutter and fibrosis along the talofibular articulation; syndesmosis
chondromalacia of the talus and ankle also may be found. In patients with anterior soft-tissue
impingement, approximately 84% who have a poor response to nonsurgical management will
have a good to excellent response after arthroscopic synovectomy and debridement.
 Ferkel RD, Fasulo GJ: Arthroscopic treatment of ankle injuries. Orthop Clin North Am 1994;25:17-32.
 Ferkel RD, Karzel RP, Del Pizzo W, Friedman MJ, Fischer SP: Arthroscopic treatment of anterolateral impingement of the
ankle. Am J Sports Med 1991;19:440-446.

78. Which of the following tissues used for anterior cruciate ligament (ACL)
reconstruction has the highest maximum load to failure?
1- Allograft ACL
2- Bone-patellar tendon-bone with a width of 10 mm
3- Fascia lata with a width of 15 mm
4- Central quadriceps tendon with a width of 15 mm
5- Quadruple semitendinosus and gracilis tendons

PREFERRED RESPONSE: 5

DISCUSSION : While the patellar tendon ligament is considered by many to be the tissue of
choice for ACL reconstruction, more recent studies have shown that the quadruple
semitendinosus and gracilis tendon graft has the greatest stiffness and offers the highest
maximum load to failure.
 Hamner DL, Brown CH Jr, Steiner ME, Hecker AT, Hayes WC: Hamstring tendon grafts for reconstruction of the anterior
cruciate ligament: Biomechanical evaluation of the use of multiple strands and tensioning techniques. J Bone Joint Surg Am
1999;81:549-557.
 Cooper DE, Deng XH, Burstein AL, Warren RF: The strength of the central third patellar tendon graft: A biomechanical
study. Am J Sports Med 1993;21:8l8-823.
 Brown CH Jr, Steiner ME, Carson EW: The use of hamstring tendons for anterior cruciate ligament reconstruction:
Technique and results. Clin Sports Med 1993;12:723-756.
 Engebretsen L, Lewis JL: Graft selection and biomechanical considerations in ACL reconstruction. Sports Med
Arthroscopy Rev 1996;4:336-341.

79. Creatine is currently being used by athletes as a dietary supplement in an attempt to


enhance performance. What is the physiologic basis for its use?
American Academy of Orthopaedic Surgeons
2001 Sports Medicine Self-Assessment Examination

1- Assists in carbohydrate metabolism and glycogen synthesis by producing


adenosine diphosphate (ADP) to enhance aerobic activities
2- Converts to phosphocreatine (PCr), which acts as an energy reservoir for
adenosine triphosphate (ATP) in muscle tissue
3- Converts to PCr, which enhances the production of ADP and promotes the
metabolism of triglycerides as an energy source
4- Converts to PCr, which enhances the production of ADP and promotes the
metabolism of proteins as an energy source
5- Converts to ADP by creatine kinase (CK) providing an energy reservoir for the
production of ATP

PREFERRED RESPONSE: 2

DISCUSSION : Creatine is currently used as a nutritional supplement in an attempt to enhance


athletic performance. The physiologic basis for its use is based on its conversion by CK to PCr,
which acts as an energy reservoir in muscle cells for the production of ATP. A number of
studies that examined the effect of creatine supplementation on performance concluded that
while creatine does not increase peak force production, it can increase the amount of work done
in the first few anaerobic short duration, maximal effort trials. The mechanism for this
enhancement of work is unknown, but it is most likely secondary to the increase in the available
PCr pool.
 Greenhaff PL: Creatine and its application as an ergogenic aid. Int J Sport Nutr 1995;5:S100-S110.
 Greenhaff PL, Casey A, Short AH, Harris R, Soderlund K, Hultman E: Influence of oral creatine supplementation on muscle
torque during repeated bouts of maximal voluntary exercise in man. Clin Sci 1993;84:565-571.
 Trump ME, Heigenhauser GJ, Putman CT, Spriet LL: Importance of muscle phosphocreatine during intermittent maximal
cycling. J Appl Physiol 1996;80:1574-1580.
 Hultman E, Soderlund K, Timmons JA, Cederblad G, Greenhaff PL: Muscle creatine loading in men. J Appl Physiol
1996;81:232-237.

80. A 16-year-old high school football player sustains an injury to the left hip. The
avulsed fragment identified by the arrow in Figure 34 represents the origin of which
of the following structures?

1- Ischiofemoral ligament.
2- Pubofemoral ligament
3- Rectus femoris
4- Sartorius
5- Gluteus minimus

PREFERRED RESPONSE: 3

DISCUSSION : The avulsed fragment represents the origin of the rectus femoris from the
anterior inferior iliac spine and the brim of the acetabulum. Avulsion of the anterior inferior iliac
Copyright©2001 by American Academy of Orthopaedic Surgeons™. All rights reserved.
84  American Academy of Orthopaedic Surgeons

spine is much less common than avulsion of the anterior superior iliac spine with its origin of the
sartorius. The origin of the gluteus minimus is from the outer cortex of the iliac wing and has
not been reported as a source of bony avulsion. The hip capsule is composed of the
ischiofemoral and pubofemoral ligaments, in addition to the iliofemoral ligament. The pelvic
attachment of the ischiofemoral ligament is from the ischial part of the acetabulum posteriorly,
while the pubofemoral ligament attaches to the pubic portion inferiorly. Technically, ligaments
do not have origins and insertions as muscle tendon groups do, but have attachment sites.
 Metzmaker JN, Pappas AM: Avulsion fractures of the pelvis. Am J Sports Med 1985;13:349-358.
 Mader TJ: Avulsion of the rectus femoris tendon: An unusual type of pelvic fracture. Pediatr Emerg Care 1990;6:198-199.

81. Which of the following methods of meniscal repair has the highest load to failure
strength?
1- Horizontal suture
2- Vertical suture
3- Mulberry knot
4- T-fix suture
5- Meniscal arrow

PREFERRED RESPONSE: 2

DISCUSSION : Numerous experimental studies have shown that vertical suture techniques are
superior to all of the other noted methods. In fact, vertical sutures have been shown to be twice
as strong as several of these techniques.
 DeHaven KE: Meniscus repair. Am J Sports Med 1999;27:242-250.
 Dervin GF, Downing KJ, Keene GC, McBride DG: Failure strengths of suture versus biodegradable arrow for meniscal
repair: An in vitro study. Arthroscopy 1997;13:296-300.
 Barber FA: Endoscopic meniscal repair: The T-fix technique. Sports Med Arthroscopy Rev 1999;7:28-33.

82. Figure 35 shows the lateral radiograph of a 15-year-old basketball player who felt a
dramatic pop in his knee when landing after a lay-up. The patient reports that he
cannot bear weight on the injured extremity. Management should consist of

1- closed reduction and casting in extension.


2- open reduction with suture fixation of the proximal
fragment.
3- closed reduction, followed by functional bracing.
4- open reduction and internal fixation with screws
and complete proximal tibial epiphysiodesis.
5- open reduction and internal fixation with screws.

PREFERRED RESPONSE: 5
American Academy of Orthopaedic Surgeons
2001 Sports Medicine Self-Assessment Examination

DISCUSSION : Tibial tubercle avulsion is an injury of the adolescent knee that most often
occurs just before the end of growth. The fracture usually occurs with jumping, either at push-
off or landing. This patient has a type III injury. In type III injuries, the articular surface is
disrupted, and meniscal injury and compartment syndrome can occur. Open reduction is the
treatment of choice, and anterior fasciotomy should be considered prophylactically at the time of
surgery. Although the fracture heals with an anterior epiphysiodesis of the proximal tibia, little
growth remains in this patient and no special handling of the physis is warranted.

 Ogden JA, Tross RB, Murphy MJ: Fractures of the tibial tuberosity in adolescents. J Bone Joint Surg Am 1980;62:205-215.
 Pape JM, Goulet JA, Hensinger RN: Compartment syndrome complicating tibial tubercle avulsion. Clin Orthop
1993;295:201-204.

83. A 52-year-old man has pain in the sternal area after landing on his right shoulder in
a fall from his bicycle. In addition, he reports that he had difficulty swallowing and
breathing immediately after the fall, but the symptoms resolved. A CT scan reveals
a posterior sternoclavicular dislocation. Initial management should include
1- a snug figure-of-8 splint and observation for spontaneous reduction.
2- closed reduction under general anesthesia.
3- closed reduction under general anesthesia and percutaneous pinning.
4- open reduction and capsuloligamentous repair.
5- open reduction and wire stabilization of the joint.

PREFERRED RESPONSE: 2

DISCUSSION : Posterior sternoclavicular dislocations require rapid treatment because of the


proximity of major neurovascular structures and the airway. Initial management should consist
of closed reduction under general anesthesia in the operating room with a chest surgeon
available. A successful closed reduction is usually stable. Internal fixation of sternoclavicular
injuries should be avoided because of the likelihood of hardware migration and possible injury to
the mediastinal structures. If closed reduction is unsuccessful, open reduction is indicated.
Treatment following reduction of the sternoclavicular joint includes the application of a figure-
of-8 splint and a sling for 6 weeks, followed by stretching and strengthening exercises.

 Beaty JH (ed): Orthopaedic Knowledge Update 6. Rosemont IL, American Academy of Orthopaedic Surgeons, 1999, pp
287-297.
 Rockwood CA Jr: Disorders of the sternoclavicular joint, in Rockwood CA Jr, Matsen FA III (eds): The Shoulder.
Philadelphia, PA, WB Saunders, 1990, vol 2, pp 1010-1017.

84. What nerve is at greatest risk of harm from the portal


shown in Figure 36?

1- Radial
2- Ulnar
3- Median
4- Lateral antebrachial cutaneous
Copyright©2001 by American Academy of Orthopaedic Surgeons™. All rights reserved.
86  American Academy of Orthopaedic Surgeons

5- Posterior antebrachial cutaneous

PREFERRED RESPONSE: 1

DISCUSSION : The figure shows the anterolateral portal for elbow arthroscopy, and injury to the
radial nerve has been reported in conjunction with this portal site. Studies have shown that
closer proximity to the radial nerve is associated with more distal portal sites. The lateral and
posterior antebrachial cutaneous nerves are both at less risk of injury. The ulnar and median
nerves are both fairly remote to this location.

 Field LD, Altchek DW, Warren RF, O'Brien SJ, Skyhar MJ, Wickiewicz TL: Arthroscopic anatomy of the lateral elbow: A
comparison of three portals. Arthroscopy 1994;10:602-607.
 Papilion JD, Neff RS, Shall LM: Compression neuropathy of the radial nerve as a complication of elbow arthroscopy: A
case report and review of the literature. Arthroscopy 1988;4:284-286.
 Poehling GG, Whipple TL, Sisco L, Goldman B: Elbow arthroscopy: A new technique. Arthroscopy 1989;5:222-224.

85. In the majority of patients with chronic anterior cruciate ligament (ACL)-deficient
knees, analysis of the gait pattern during level walking will most likely reveal which
of the following changes?
1- No significant differences in gait from the contralateral knee
2- No change in knee flexion-extension moment with balancing of quadriceps and
hamstring activity
3- A change in knee flexion-extension moment with decreased hamstring activity
4- A change in knee flexion-extension moment with decreased demand on the
quadriceps and a net increase in hamstring activity
5- A change in knee flexion-extension moment with increased demand on the
quadriceps

PREFERRED RESPONSE: 4

DISCUSSION : Patients with chronic ACL-deficient knees typically have lower than normal net
quadriceps activity during the middle portion of the stance phase; the net moment about the knee
reverses from one that demands quadriceps activity to one that demands increased hamstring
activity. This type of gait is termed "quadriceps avoidance." This avoidance is believed to be a
functional adaptation to reduce anterior tibial translation, and it is most prevalent as the knee
moves from 45 degrees of flexion toward full extension, the arc of motion through which the
ACL is most responsible for stability.

 Hurwitz DE, Andriacchi TP, Bush-Joseph CA, Bach BR Jr: Functional adaptations in patients with ACL-deficient knees.
Exerc Sport Sci Rev 1997;25:1-20.
 Andriacchi TP, Birac D: Functional testing in the anterior cruciate ligament-deficient knee. Clin Orthop 1993;288:40-47.
 Solomonow M, Baratta R, Zhou BH, et al: The synergistic action of the anterior cruciate ligament and thigh muscles in
maintaining joint stability. Am J Sports Med 1987;15:207-213.
American Academy of Orthopaedic Surgeons
2001 Sports Medicine Self-Assessment Examination

86. Glenohumeral inferior stability in the adducted shoulder position is primarily a


function of the
1- rotator cuff.
2- posterior glenohumeral ligament.
3- long head of the biceps tendon.
4- inferior glenohumeral ligament complex.
5- superior glenohumeral ligament.

PREFERRED RESPONSE: 5

DISCUSSION : When the arm is adducted, the superior structures, including the superior
glenohumeral ligament, are responsible in limiting the inferior translation. With the arm
abducted, the inferior glenohumeral ligament complex is responsible for limiting inferior
subluxation. Rotator cuff activity can actually depress the humeral head and does not play a role
in preventing inferior subluxation. The long head of the biceps and the posterior glenohumeral
ligament do not play a role in protecting the shoulder from inferior instability.

 Warner JJ, Deng XH, Warren RF, Torzilli PA: Static capsuloligamentous restraints to superior-inferior translation of the
glenohumeral joint. Am J Sports Med 1992;20:675-685.
 Harryman DT II, Sidles JA, Harris SL, Matsen FA III: The role of the rotator interval capsule in passive motion and stability
of the shoulder. J Bone Joint Surg Am 1992;74:53-66.

87. A 20-year-old football player has repeated episodes of heat cramps during summer
training sessions. A deficiency of what electrolyte is most responsible for heat
cramps?
1- Potassium
2- Magnesium
3- Chloride
4- Sodium
5- Iron

PREFERRED RESPONSE :4

DISCUSSION : Sodium deficiency is the cause of heat cramps. It is the principle electrolyte of
sweat and is readily lost during training, especially in warmer temperatures. The condition can
be avoided by adding extra table salt to food and maintaining good hydration before and after
sports activities. Salt tablets are to be avoided when a patient has heat cramps because the high
soluble load will cause gastric irritation

 Bergeron MF, Armstrong LE, Maresh CM: Fluid and electrolyte losses during tennis in the heat. Clin Sports Med
1995;14:23-32.
 Halpern B: Fluid and electrolyte replacement in athletes. Sports Med Digest 1994;16:1-5.

88. Figure 37 shows the radiograph of a 23-year-old football player who sustained a
blow to the anterior aspect of his shoulder. Examination reveals pain and limited
Copyright©2001 by American Academy of Orthopaedic Surgeons™. All rights reserved.
88  American Academy of Orthopaedic Surgeons

rotation. He is unable to flex the arm above the shoulder. Management should
include which of the following studies?

1- Axillary radiograph
2- Arthrogram
3- Electromyogram
4- Bone scan
5- Arteriogram

PREFERRED RESPONSE :1

DISCUSSION : The patient has a posterior dislocation. The radiograph reveals marked internal
rotation, but fails to show whether the humeral head is posteriorly displaced. Therefore, an
axillary radiograph should be obtained to help confirm the diagnosis. Transverse view CT or
MRI scans also may be useful. The other studies will not help confirm the diagnosis. In addition
to a direct posterior blow, a shoulder dislocation may be caused by a seizure disorder or
electrocution.

 Bloom MH, Obata WG: Diagnosis of posterior dislocation of the shoulder with the use of Velpeau axillary and angle-up
roentgenographic views. J Bone Joint Surg Am 1967;49:943-949.
 Rockwood CA: Subluxations and dislocations about the shoulder, in Rockwood CA, Green DP (eds): Fractures in Adults, ed
2. Philadelphia, PA, JB Lippincott, 1984, vol 1, pp 806-856.

89. A 21-year-old football player had severe pain and immediate swelling in the left
anteromedial chest wall while bench pressing near maximal weights several days
ago. Examination at the time of injury revealed a mass on the anteromedial chest
wall. Follow-up examination now reveals decreased swelling, and axillary webbing
is observed. The patient has weakness to adduction and forward flexion. The
injured muscle originates from the
1- proximal clavicle and sternocostal margin.
2- proximal humerus.
3- coracoid process.
4- distal clavicle and acromion.
5- anterior scapula.

PREFERRED RESPONSE :1

DISCUSSION : The patient has a pectoralis major rupture, an injury that occurs most commonly
during weight lifting. Grade III injuries represent complete tears of either the musculotendinous
junction or an avulsion of the tendon from the humerus, the most common injury site.
American Academy of Orthopaedic Surgeons
2001 Sports Medicine Self-Assessment Examination

Examination will most likely reveal ecchymoses and swelling in the proximal arm and axilla,
and strength testing will show weakness with internal rotation and in adduction and forward
flexion. Axillary webbing, caused by a more defined inferior margin of the anterior deltoid as
the result of rupture of the pectoralis, can be seen as the swelling diminishes. Surgical repair is
the treatment of choice for complete ruptures. Nonsurgical treatment is associated with
significant losses in adduction, flexion, internal rotation, strength, and peak torque. The
pectoralis major originates from the proximal clavicle and the border of the sternum, including
ribs two through six. The pectoralis major inserts (rather than originates) on the humerus. The
coracoid process is the insertion site for the pectoralis minor, as well as the origin for the
conjoined tendon. The pectoralis major has no attachment or origin from the scapula. The
anterior deltoid originates from the lateral one third of the clavicle and the anterior acromion.

 Miller MD, Johnson DL, Fu FH, Thaete FL, Blanc RO: Rupture of the pectoralis major muscle in a collegiate football
player: Use of magnetic resonance imaging in early diagnosis. Am J Sports Med 1993;21:475-477.
 Wolfe SW, Wickiewicz TL, Cavanaugh JT: Ruptures of the pectoralis major muscle: An anatomic and clinical analysis.
Am J Sports Med 1992;20:587-593.

90. A 24-year-old baseball pitcher reports pain over the posterior aspect of his shoulder
that occurs only during throwing. He notes that the discomfort is greatest during
the late cocking and early acceleration phases. Examination reveals localized
tenderness with palpation over the external rotators and posterior glenoid.
Radiographs are shown in Figures 38a through 38c. What is the most likely
diagnosis?

1- Fracture of the posterior glenoid


2- Triceps insertion avulsion
3- Calcific tendinitis
4- Posterior glenoid exostosis
5- Loose body

PREFERRED RESPONSE :4
Copyright©2001 by American Academy of Orthopaedic Surgeons™. All rights reserved.
90  American Academy of Orthopaedic Surgeons

DISCUSSION : The radiographs show a posterior glenoid osteophyte, often termed a "thrower's
exostosis." These exostoses are best visualized on the Stryker notch view and may be missed on
other more standard radiographic views of the shoulder. CT and MRI scans may be used, but
usually add little information to the radiographic findings. Arthroscopic examination of patients
with this condition commonly reveals undersurface tearing of the rotator cuff and posterior
labrum. Treatment of this condition remains somewhat controversial, with avocation of both
nonsurgical and surgical techniques.

 Meister K, Andrews JR, Batts J, Wilk K, Baumgarten T, Baumgartner T: Symptomatic thrower's exostosis: Arthroscopic
evaluation and treatment. Am J Sports Med 1999;27:133-136.
 Ferrari JD, Ferrari DA, Coumas J, Pappas AM: Posterior ossification of the shoulder: The Bennett lesion. Etiology,
diagnosis, and treatment. Am J Sports Med 1994;22:171-176.
 Walch G, Boileau P, Noel E, et al: Impingement of the deep surface of the supraspinatus tendon on the posteriorsuperior
glenoid rim: An arthroscopic study. J Shoulder Elbow Surg 1992;1:238-245.

91. What percent of the adult human meniscus is vascularized?


1- 0%
2- 5%
3- 25%
4- 50%
5- 100%

PREFERRED RESPONSE :3

DISCUSSION : The adult menisci are considered to be relatively avascular structures, with the
peripheral blood supply originating predominately from the lateral and medial genicular arteries.
Branches of these vessels form the perimeniscal capillary plexus, which supplies the peripheral
border throughout its attachment to the joint capsule. Vascular penetration studies have shown
that 10% to 30% of the peripheral portion of the medial meniscus and 10% to 25% of the lateral
meniscus are vascularized.

 Arnoczky SP, Warren RF: Microvasculature of the human meniscus. Am J Sports Med 1982;10:90-95.
 Arnoczky SP, Warren RF: The microvasculature of the meniscus and its response to injury: An experimental study in the
dog. Am J Sports Med 1983;11:131-141.

92. A 30-year-old man who participates in recreational sports reports the spontaneous
onset of intermittent pain and swelling about the right knee. Examination reveals a
3+ effusion, with a range of motion of 10 degrees to 60 degrees. He has mild diffuse
tenderness but no instability. MRI scans and an arthroscopic view are shown in
Figures 39a through 39c. Management should consist of
1- arthroscopic debridement of the articular lesion and resurfacing.
2- knee aspiration and an intra-articular cortisone injection.
3- rheumatologic evaluation.
4- infectious disease evaluation for possible Lyme disease.
5- arthroscopic synovectomy.
American Academy of Orthopaedic Surgeons
2001 Sports Medicine Self-Assessment Examination

PREFERRED RESPONSE :5

DISCUSSION : The patient has synovial chondromatosis. The MRI scans show multiple small
proscribed areas of signal intensity in the gutters and suprapatellar pouch, suggesting very small
loose bodies. The arthroscopic view shows the classic appearance of multiple small chondral
loose bodies. Synovial chondromatosis is a condition in which the synovium undergoes
metaplasia, producing multiple chondral loose bodies that can subsequently ossify. The
treatment of choice, removal of the loose bodies and arthroscopic synovectomy, results in a
lower incidence of recurrence than other treatment methods.

 Coolican MR, Dandy DJ: Arthroscopic management of synovial chondromatosis of the knee: findings and results in 18
cases. J Bone Joint Surg Br 1989;71:498-500.
 Ogilvie-Harris DJ, Saleh K: Generalized synovial chondromatosis of the knee: A comparison of removal of the loose bodies
alone with arthroscopic synovectomy. Arthroscopy 1994;10:166-170.

93. Figure 40 shows the plain radiograph of a 30-year-old woman who has had a long
history of standing bilateral anterior knee pain and a sense of patellar instability
without frank dislocation. Nonsurgical management consisting of anti-
inflammatory drugs and physical therapy has failed to provide relief. Examination
reveals full range of motion of both knees, with moderate patellofemoral crepitance.
Patellar apprehension and patellar grind tests are positive. The Q-angle measures
20 degrees. Management should now consist of

1- bilateral arthroscopic lateral


releases.
2- bilateral arthroscopic lateral
releases and medial retinacular
thermal shrinkage.
3- bilateral lateral releases and
anteromedialization of the
tibial tubercles.
4- physical therapy and the use of
patella-stabilizing braces.
5- physical therapy with taping.

Copyright©2001 by American Academy of Orthopaedic Surgeons™. All rights reserved.


92  American Academy of Orthopaedic Surgeons

PREFERRED RESPONSE :3

DISCUSSION : The history, physical examination, and radiographs indicate that the
patellofemoral pain is most likely caused by excessive lateral patellar pressure and patellar
maltracking. Because the radiographs reveal the lateral tilt of the patella and lateral subluxation,
the treatment of choice is bilateral lateral releases with anteromedialization of the tibial tubercles.
This procedure corrects not only the excessive lateral patellar pressure, but also the lateral
subluxation. The use of patella-stabilizing braces or taping may provide temporary relief, but
these implements are not well-tolerated and they will not change the underlying biomechanics of
the knee. Simple lateral release is indicated for isolated lateral tilt, but it does not correct the
lateral subluxation. The use of thermal capsular shrinkage for the medial retinaculum has not
been proven to provide long-term correction of the deformity.

 Boden BP, Pearsall AW, Garrett We Jr, et al: Patellofemoral instability: Evaluation and management. J Am Acad Orthop
Surg 1997;5:47-57.
 Fulkerson JP: Patellofemoral pain disorders: Evaluation and management. J Am Acad Orthop Surg 1994;2:124-132.

94. A 24-year-old runner who underwent an allograft reconstruction of the anterior


cruciate ligament (ACL) 3 years ago now reports anterior knee pain. Examination
reveals no swelling or effusion, and the patient has full motion. A Lachman test and
a pivot-shift test are negative. Palpation reveals tenderness on the patellar tendon
and at the inferior pole of the patella. AP and lateral radiographs are shown in
Figures 41a and 41b. Management should consist of

1- immediate biopsy of the


proximal tibia.
2- aspiration and culture of
the knee. 3-
3- observation with activity
modification.
4- a white blood cell scan.
5- revision of the ACL
reconstruction.

PREFERRED RESPONSE :3

DISCUSSION : The radiographs show tunnel enlargement, which is seen after ACL
reconstruction, particularly with allografts. Occasionally, there will be formation of an
associated subcutaneous pretibial cyst. It has been proposed that the tunnel enlargement and cyst
are the result of incomplete incorporation of allograft tissues within the bone tunnels. There may
be residual graft necrosis, allowing synovial fluid to be transmitted through the tunnel to collect
in the pretibial area, manifesting as a synovial cyst. In the absence of cyst formation, the
presence of tunnel enlargement does not appear to adversely affect the clinical outcome. Based
American Academy of Orthopaedic Surgeons
2001 Sports Medicine Self-Assessment Examination

on studies by Fahey and associates, continued tunnel expansion does not occur. Victoroff and
associates report good results with curettage and bone grafting of the tibial tunnel if a pretibial
cyst is present. Because this patient does not have a pretibial cyst, observation with activity
modification is the preferred treatment.

 Fahey M, Indelicato PA: Bone tunnel enlargement after anterior cruciate ligament replacement. Am J Sports Med
1994;22:410-414.
 Victoroff BN, Paulos L, Beck C, Goodfellow DB: Subcutaneous pretibial cyst formation associated with anterior cruciate
ligament allografts: A report of four cases and literature review. Arthroscopy 1995;11:486-494.

95. What is the most common mechanism of injury that produces turf toe?
1- Valgus stress at the first metatarsophalangeal (MTP) joint
2- Hyperflexion stress
3- Hyperextension stress
4- Varus stress
5- Axial load

PREFERRED RESPONSE :3

DISCUSSION : The most common mechanism of injury for turf toe is a hyperextension injury to
the MTP joint. The foot is typically in a dorsiflexed position with the heel raised when an
external force drives the MTP joint into further dorsiflexion. The joint capsule usually tears at
the metatarsal neck because its attachment is weaker there than at the proximal phalanx. Some
compression injuries to the dorsal articular surface of the metatarsal head can result from
extension or hyperextension.

 Clanton TO, Ford JJ: Turf toe injury. Clin Sports Med 1994;13:731-741.
 Rodeo SA, O'Brien S, Warren RF, et al: Turf toe: An analysis of metatarsophalangeal joint sprains in professional football
players. Am J Sports Med 1990;18:280-285.

96. A 68-year-old man embarks on a 24-week strength training program. He trains at


80% of his single repetition maximum for both the upper and lower extremities.
Which of the following changes can be anticipated?
1- An absolute decrease in aerobic capacity
2- A decrease in capillary density in the trained muscles
3- A significant increase in strength
4- A significant improvement in Vo2max
5- No change in the cross-sectional area of the trained muscles

PREFERRED RESPONSE :3

DISCUSSION : Consistent, long-term exercise training in older athletes has proven very
beneficial in reversing both endurance and strength losses that traditionally have been seen with
aging. This patient's program will lead to a significant increase in the strength, cross-sectional
area, and capillary density of the trained muscles. No major changes in aerobic capacity are
Copyright©2001 by American Academy of Orthopaedic Surgeons™. All rights reserved.
94  American Academy of Orthopaedic Surgeons

anticipated. Strength improvements of up to 5% per day, similar to those for younger athletes,
have been identified in this population in one study.

 Kirkendall DT, Garrett WE Jr: The effects of aging and training on skeletal muscle. Am J Sports Med 1998;26:598-602.
 Frontera WR, Meredith CN, O'Reilly KP, Knuttgen HG, Evans WJ: Strength conditioning in older men: Skeletal muscle
hypertrophy and improved function. J Appl Physiol 1988;64:1038-1044.

97. A 26-year-old ballet dancer reports posterolateral ankle pain, especially with
maximal plantar flexion. Examination reveals maximal tenderness just posterior to
the lateral malleolus, and symptoms are heightened with forced passive plantar
flexion. Radiographs are shown in Figures 42a and 42b. What is the most likely
cause of the patient's symptoms?

1- Posterior impingement
of the os trigonum.
2- Subluxation of the
peroneal tendon
3- Posterior tibial stress
fracture.
4- Osteochondritis
dissecans of the lateral
dome of the talus.
5- Stenosis of the peroneal
tendon sheath

PREFERRED RESPONSE :1

DISCUSSION : The patient has a symptomatic os trigonum caused by impingement that occurs
with maximal plantar flexion of the ankle in the demi-pointe or full-pointe position. Patients
frequently report posterolateral pain localized behind the lateral malleolus that may be
misinterpreted as a disorder of the peroneal tendon. Pain with passive plantar flexion (the plantar
flexion sign) indicates posterior impingement, not a problem with the peroneal tendon. The
symptoms are not characteristic of a stress fracture, nor do the radiographs show a stress fracture
or an osteochondritis dissecans lesion. The os trigonum is modest in its dimensions. The
incidence or magnitude of symptoms does not correlate with the size of the fragment. Large
fragments may be asymptomatic, while small lesions may create significant symptoms.

 Marotta JJ, Micheli LJ: Os trigonum impingement in dancers. Am J Sports Med 1992;20:533-536.
 Hamilton WG: Foot and ankle injuries in dancers, in Mann RA, Coughlin MJ (eds): Surgery of the Foot and Ankle, ed 6. St
Louis, MO, CV Mosby, 1993, pp 1241-1276.

98. A 23-year-old soccer player sustains a grade III complete posterior cruciate
ligament (PCL) tear after colliding with another player. In reconstructing the PCL,
it is optimal to reconstruct the
American Academy of Orthopaedic Surgeons
2001 Sports Medicine Self-Assessment Examination

1- anterolateral bundle and tension the graft at 10 degrees of flexion.


2- anterolateral bundle and tension the graft at 90 degrees of flexion.
3- posteromedial bundle and tension the graft at 10 degrees of flexion.
4- posteromedial bundle and tension the graft at 45 degrees of flexion.
5- posteromedial bundle and tension the graft at 90 degrees of flexion.

PREFERRED RESPONSE :2

DISCUSSION : The PCL is a nonisometric structure with nonuniform tension during knee
motion, with maximum tension at 90 degrees of flexion. While the posteromedial PCL fibers
have been found to be the most isometric, the anterolateral fibers represent the bulk of the
ligament. Studies have suggested that anterior placement of the femoral tunnel is superior to
placement in an isometric position. The anterolateral bundle tightens as the knee flexes;
therefore, it is optimal to tension the graft at 90 degrees of flexion.

 Harner CD, Xerogeanes JW, Livesay GA, et al: The human posterior cruciate ligament complex: An interdisciplinary study.
Ligament morphology and biomechanical evaluation. Am J Sports Med 1995;23:736-745.
 Burns WC II, Draganich LF, Pyevich M, Reider B: The effect of femoral tunnel position and graft tensioning technique on
posterior laxity of the posterior cruciate ligament-reconstructed knee. Am J Sports Med 1995;23:424-430.

99. Figure 43 shows the lateral radiograph of a patient who underwent anterior cruciate
ligament reconstruction. Based on the tunnel placement shown in the radiograph,
evaluation of postoperative knee range of motion will most likely show

1- normal flexion and extension.


2- loss of extension.
3- loss of flexion.
4- loss of flexion and extension.
5- hyperextension.

PREFERRED RESPONSE :3

DISCUSSION : The radiograph shows the correct tibial tunnel and anterior femoral tunnel;
therefore, range of motion will most likely show loss of flexion.

 Brown CH Jr, Carson EW: Revision anterior cruciate ligament surgery. Clin Sports Med 1999;18:109-171.
 Brown HR, Indelicato PA: Complications of anterior cruciate ligament reconstruction. Op Tech Orthop 1992;2:125-135.

Copyright©2001 by American Academy of Orthopaedic Surgeons™. All rights reserved.


96  American Academy of Orthopaedic Surgeons

100. A 10-year-old soccer player has bilateral heel pain and reports that the pain is
worse during and immediately after sports. Examination reveals that the calcaneal
tuberosities are painful to palpation bilaterally. What is the most likely diagnosis?
1- Plantar fasciitis
2- Calcaneal apophysitis
3- Achilles tendinitis
4- Calcaneal bursitis
5- Stress fractures of the calcanei

PREFERRED RESPONSE :2

DISCUSSION : Calcaneal apophysitis (Sever's disease) is a common cause of heel pain in


children who are active in sports. The symptoms are most commonly bilateral and will often
respond to a gastrocnemius-soleus complex stretching program. In addition, rest, anti-
inflammatory drugs, and heel pads for the shoe may be prescribed. There is no effect on the
long-term growth of the calcaneus.

 Micheli LJ, Ireland ML: Prevention and management of calcaneal apophysitis in children: An overuse syndrome. J Pediatr
Orthop 1987;7:34-38.
 Bernhardt DT, Landry GL: Sports injuries in young athletes. Adv Pediatr 1995;42:465-500.

You might also like